500 Questions

December 27, 2017 | Author: dprincebuchikoy | Category: Esophagus, Bile, Peptic Ulcer, Myocardial Infarction, Systemic Lupus Erythematosus
Share Embed Donate


Short Description

medical...

Description

MCU-FDT MEDICAL FOUNDATION and ASSOCIATION OF PHILLIPINE MEDICAL COLLEGE GASTROENTEROLOGY : DR. QUINTIN BABARAN 1. Mark, a 7 yr old boy was passing by McDonald’s and smelled the aroma of hamburgers. At this instant, acid secretion took place in his stomach. Which phase of acid secretion was responsible for the acid secretion? Cephalic C. Intestinal A. Gastric D. Interdigestive B. 2. While Mark was eating his hamburger, the food in his stomach stimulated further acid secretion. Which of the following substances is responsible for the increased acid secretion? Histamine C. Somatostatin A. Gastrin D. Acetylcholine B. 3. Mark grew up to become an executive of a leading company. Because of the stresses of his job, he developed Duodenal Ulcers. The ulcers of Mark will be found: 3cm from the pyloric opening C. 2nd portion of the duodenum A. 10cm from the duodenal bulb D. 3rd potion of the duodenum B. 4. Mark consulted his physician at MCU because of his Duodenal ulcer. The physician examined Mark and found that Mark had a succussion splash. This finding indicates that Mark had this DU complication. Perforation C. Gastric outlet obstruction A. Penetration D. Hemorrhage B. 5. Which of the following items below is a function of the pre-epithelial level of defense to prevent mucosal damage? The mucus gel functions as a unstirred water layer impeding diffusion of A. ions and molecules including pepsin The surface epithelial cells act as ionic transporters that maintain B. intracellular pH and bicarbonate production Gastric epithelial cells bordering an injury migrate to restore a damage C. region Epithelial cell renewal occurs and together with angiogenesis preserve D. integrity of the gastric tissues 6. Mr. Ulcero is diagnosed to have a malignant gastric ulcer by double contrast radiography. Which of the following findings does not confirm a malignant ulcer ? Large Ulcer greater than 3cm A. An ulcer on top of a mass B. Gastric folds that radiate up to the margin of the ulcer C. Gatric ulcers occurring in the antrum in the presence of duodenal ulcers D. 7. Which of the following is not a manifestation of gastrointestinal hemorrhage? Occult blood in the stools C. Hemoptysis A. Hematemesis D. Pallor B. 8. Mr. Norman had an upper G.I. hemorrhage. All of the following can be a differential diagnosis EXCEPT: Meckel’s diverticulum C. Duodenal ulcer A. Esophageal Varices D. Mallory-Weiss tear B. 9. The goals of treatment of patients with peptic ulcer include all of the following EXCEPT: Relieve the pain C. Prevent ulcer recurrence A. Promote ulcer healing D. Reduce the functional parietal cell B.

10. A patient who suddenly develops severe abdominal pain with board-like rigidity of the abdomen suggests the occurrence of which complication of PUD? Gastric outlet obstruction C. Perforation A. Penetration D. Distention B. 11. Duodenal ulcers commonly penetrate which organ? Pancreas C. Liver A. Spleen D. Right kidney B. 12. All of the following conditions can cause reflux, EXCEPT: After meals C. Pregnancy A. Bending position D. Upright position B. 13. Gastroesophageal reflux can be diagnosed byany of the following EXCEPT: Barium contrast radiography C. Endoscopy A. Plain radiography D. Bernstein’s test B. 14. Constipation that is not associated with structural abnormality of the colon is seen in: Colon cancer C. Post-hysterectomy injury to rectum A. Idiopathic constipation D. Rectal polyp B. 15. Carcinoma arising from a Barrett’s esophagus can be suspected when: Prolonged dysphagia is present A. Rapid dysphagia develops together with weight loss B. Heartburn intensifies C. Hematemesis occurs D. 16. Peptic stricture of the esophagus should be suspected when one of the following symptoms is present in reflux esophagitis: Vomiting C. Weight loss A. Odynophagia D. Dysphagia B. 17. Change in the mucosal lining of the distal esophagus from squamous to columnar cells is seen in one of the following conditions: Hiatal hernia C. Barrett’s esophagus A. Esophageal web D. Esophageal carcinoma B. 18. Proton pump inhibitors achieves one of the following goals in the treatment of gastroesophageal reflux disease: Decreases gastroesophageal reflux A. Renders the refluxate harmless B. Improves esophageal clearance C. Protects the esophageal mucosa D. 19. The anti-reflux surgery is performed by: Wrapping the gastric fundus around the esophagus A. Resecting the gastric fundus and anastomose it with the esophagus B. Resecting both the lower esophagus and fundus and do anastomosis C. Wrapping the esophagus around the gastric fundus D. 20. The classic manifestation of reflux is: Heartburn A. Dysphagia B.

C. Odynophagia D. Angina

21. Which of the following is not a major composition of Bile? Water 82% C. Conjugated bilirubin A. Bile acids D. Lecithin B. 22. The basal daily secretion of bile is in milliliters is: 500-600 C. 800-900 A. 200-300 D. 1000-1200 B.

23. Cholic and chenodeoxycholic acid are the primary bile acids. They are formed: In the liver from the synthesis of cholesterol. A. In the colon as a bacterial metabolite of the primary bile acids. B. In the colon due to poor absorption of deoxicholic acid. C. In the bile canaliculi before secretion in the bile ducts. D. 24. Which of the following is not a function of the bile acids? Facilitates biliary excretion of cholesterol A. Necessary for the normal intestinal absorption of dietary fats via a micellar B. transport mechanism Serve as a major physiologic driving force for hepatic bile flow and aid in C. water and electrolyte transport in the small bowel and colon. Blocks the reabsorption of cholesterol from the ileum D. 25. The reabsorption of bile acids that enter the portal bloodstream and are taken up rapidly by hepatocytes, reconjugated, and resecreted into bile is known as: Bile conjugation C. Bile excretion A. Enterohepatic circulation D. Biliary secretion B. 26. Which of the following statements is not true regarding bile acids? The normal bile acid pool size is approximately 2 to 4 g. A. The bile acid pool undergoes at least one or more enterohepatic cycles B. 95% percent of the bile acids are excreted with the stools after the first C. pass The bile acid pool circulates approximately 5 to 10 times daily D. 27. During a meal, the sphincter of Oddi is: Tightly closed to prevent the reflux of duodenal content into the bile and A. pancreatic ducts Tonically contracted to allow gallbladder filling B. Closed to offer a high pressure zone of resistance to bile flow from the C. common bile duct to the duodenum Open to allow the flow of bile from the common bile ducts to the D. duodenum to aid in digestion. 28. During a meal, which of the following hormones is secreted to stimulate gallbladder emptying? Secretin C. Cholecystokinin A. Pepsin D. Somatostatin B. 29. Sarah, a 37yr old mother experienced a severe RUQ colicky pain. The best diagnostic test to confirm the presence of a gallstone is: Plain abdominal x-ray A. Ultrasonography B. Oral cholecystography C. Radiopharmaceutical imaging (HIDA, DIDA, etc.) D. 30. Deep inspiration or cough during subcostal palpation of the RUQ usually produces increased pain and inspiratory arrest. This phenomenon is known as: Charcot’s triad C. Caroli’s syndrome A. Courvoisier’s law D. Murphy’s sign B. 31. Which of the following stimulates secretin secretion? Gastric acid C. Somatostatin A. Cholecystokinin D. Pepsin B. 32. How much of isososmotic alkaline fluid does the pancreas normally secrete in one day? 1 – 2 liters C. 500cc to 800cc A. 1.5 – 3 liters D. 4 – 5 liters B.

33. Which hormone evoke the secretion of enzyme-rich fluid? Somatostatin C. Gastrin A. Cholecystokinin D. Pepsin B. 34. Among the different electrolytes secreted by the pancreas, which has a primary physiologic importance? Sodium C. Chloride A. Calcium D. Bicarbonates B. 35. Which of the following will establish the diagnosis of acute pancreatitis? Leukocytosis of >15,000 cells/microliter A. Hyperbilirubinemia of > 68mmols/L B. Amylase > 920 I.U./L C. Hyperglycemia > 18 mmols/L D. 36. A mass usually noted with ultrasonography that contains collections of tissue, fluid, debris, pancreatic enzymes, and blood which develop over a period of 1 to 4 weeks after the onset of acute pancreatitis is known as: Pancreatic abscess C. Phlegmon A. Pancreatic pseudocyst D. Pseudocyesis B. 37. The diagnosis of pancreatic insufficiency can be best demonstrated by which of the following test? CT scan C. Secretin test A. Ultrsonography D. ERCP B. 38. Which of the following vitamins is deficient in association with malabsorption due to excessive binding of cobalamin to cobalamin-binding proteins other than intrinsic factor Vitamin B12 C. Vitamin B1 A. Vitamin B6 D. Vitamin B3 B. 39. The primary treatment of malabsorption due to chronic pancreatitis is: Total pancreatectomy C. Secretin replacement A. Sphincterotomy D. Enzyme replacement B. 40. The primary pathophysiology of alcohol-induced chronic pancreatitis is: Precipitation of protein (inspissated enzymes) in the pancreatic ducts A. Direct injury of the pancreas by the alcohol B. Compromise of the vascular supply C. Activation of the enzyme by contact with alcohol D. 41. R Domingo had a binge of alcohol 7 days pta. He came to the ER because of persistent Severe abdominal pain that started 7 days PTA. A diagnosis of uncomplicated acute pancreatitis was entertained. Which of the following test results would be expected: Normal amylase;Normal Lipase A. Elevated amylase; elevated lipase B. Normal amylase, elevated lipase C. Elevated amylase; normal lipase D. 42. In acute pancreatitis, a normal serum amylase level may be obtained if: The serum specimen is taken more than 5 days after the acute attack A. There is hypertriglyceridemia B. There is extensive pancreatic necrosis C. Incomplete ductal obstruction D. 43. A test of exocrine pancreatic function, which indirectly reflects intraluminal chymotrypsin activity is: Bentiromide test C. Secretin Test A. Bernstein test D. Edrophonium test B.

44. Auto digestion of the pancreas is prevented by: Packaging of proteases in precursor form and by the synthesis of protease A. inhibitors Secretion of somatostatin that prevents secretion of active pancreatic B. enzymes Negative feedback mechanism induced by the presence of active serine C. proteases in the duodenum Serine proteases inhibit pancreatic secretion by acting on a CCK-releasing D. peptide in the lumen of the small intestine 45. Which of the following conditions can precipitate repeated attacks of acute pancreatitis? Hyperuricemia C. Hyperglycemia A. Hypertriglyceridemia D. hypercholesterolemia B. 46. The serologic marker that provides a convenient, readily detectable, qualitative marker of HBV replication and relative infectivity is: HBsAg C. Anti HBe A. HBeAg D. Anti HBc B. 47. A 28 yr old pregnant woman consulted because of a finding of Reactive HBsAg during a routine prenatal check-up. Further serologic testing showed reactive HBeAg. The interpretation of the serologic test is that the mother’s serum: Is highly infectious and associated with the presence of hepatitis B visions A. Will rarely (10 to 15 percent) infect their offspring B. Shows harbinger of clinical improvement and resolution of infection C. Indicates a serious clinical course D. 48. A surgeon who was performing an emergency appendectomy on a HBsAg Reactive 18 yr old patient accidentally cut himself on the fingers during the course of the operation. In the event that the surgeon will be infected with hepatitis B, the serologic abnormality that appears first is: Elevated ALT C. Reactivity to HBeAg A. Reactivity to HBsAg D. Elevated Bilirubin B. 49. In acute viral hepatitis A, which of the following DOES NOT occur when jaundice appears: Persisence of virus in the liver A. Diminished viral shedding in the stools B. Increased infectivity C. Diminished viremia D. A 65 year old farmer was admitted because of early satiety, vomiting and marked weight loss. 50. Among the following differential diagnoses, your first consideration should be Duodenal ulcer with gastric outlet obstruction A. Gastric malignancy at the antrum B. Diabetic gastroparesis C. Gastric malignancy at the cardia D. NEPHRO : DR. ROMMEL TOLENTINO 51. The average, normal, kidney size is: a. 6 cm b. 8 cm c. 9 cm d. 11 cm

52. The capillary hydraulic pressure pushes the fluid from the intravascular space to the: a. Intracellular space b. Interstitial space c. Bowman’s space d. Plasma 53. Stimulation of arginine vasopressin will result in: a. Water reabsorption by the kidneys b. Diuresis c. Dehydration d. Oliguria 54. The interstitial fluid computation for a 68 kg man is: a. 6 liters b. 8 liters c. 10 liters d. 13 liters 55. Potassium is mainly found in which compartment? a. Plasma b. Interstitium c. Intracellular d. Extracellular 56. Most of the sodium is reabsorbed in which segment of the nephron: a. Distal tubules b. Proximal tubules c. Loop of Henle d. Collecting ducts 57. The kidney receives how much of the cardiac output? a. 20% b. 30% c. 40% d. 50% 58. The creatinine clearance is affected by: a. Body weight b. Urine sodium c. Exercise d. Fruits in the diet 59. The triggering factor for the release of renin is: a. Infection b. Hypoxia c. Hyponatremia d. Hypergycemia 60. The most active vitamin D metabolite that is responsible for the kidney’s participation in maintaining good quality bone is: a. D3 b. (OH) D3 c. 1, 25, (OH)2 D3 d. 25 (OH) D2

I Matching Type. Match the clinical situations in Column A with the Clinical Syndromes in Column B. The letter can be used only once. Each number is worth 1% Column A: 61. Eunice, 15 year old, female, has been suffering from infected scabies for 1 month. One morning, she noticed her eyelids to be puffy and her urine to be tea colored. She consulted a health center and her BP was recorded to be 150 / 100. Urinalysis was done and showed (+) 2 protein, 30-40 RBC, 10-15 WBC, 510 fine granular casts. E 62. 12. Mr. Cordova, 65 year old, known hypertensive, has been noticing that it takes more than 5 minutes before his urine would start to come out. And at the end of his urination, he feels that there is still significant urine left in his urinary bladder. For the last 24 hours, he estimates that he voids about 1 cup of urine. There’s a worrisome discomfort in his hypogastric area. F 63. Last summer a heat wave swept through Europe. Most of the dead people were the elderly who were left at home without companions. One of those patients who was rescued and died in the hospital was an 80 year old, female. Her BP was 70 / 30, sunken eyeballs, and dry lips. Her diaper which was not changed for 2 days was noticed to be minimally wet with urine. A 64. Aling Luisa lives with the family in a remote place in Bicol near Mayon volcano. She often feels some pain during urination which is relieved with drinking coconut juice and banaba tea. One day, the pain became severe, now accompanied by hypogastric pain, and fever. D 65. Cornelio is a 45 year old, seaman who comes home every 2 years. He was diagnosed to have high blood sugar 5 years ago during his routine medical check up. He did not mind this condition as he was asymptomatic. He would be able to take with him drugs to lower his blood sugar good for 1 month only. A month prior to his coming home for vacation, he noticed that his shoes have become tight, and his belt has shortened. His latest urinalysis showed (+) 4 albumin. B Column B a. b. c. d. e. f. g.

Acute Renal Failure Nephrotic Syndrome Interstitial Nephritis Urinary Tract Infection Acute Glomerulonephritis Obstructive Uropathy Chronic Renal Failure

II Matching Type: Match the clinical case in Column A with the part of the kidney that is involved in Column B. A letter may be used more than once. Each number is worth 1% Column A 66. Mr. Dimacali is a 50 year old, business executive, who has been hypertensive since age 45. He rarely takes his anti-hypertensive as he is very busy with his work. He had his executive check up and his doctor told him that his hypertension has already affected his kidneys. A 67. Madonna, a GRO, complains of severe terminal dysuria over a week now with accompanying suprapubic pain. C

68. Silvestre is a 49 year old, jeepney driver, who suddenly felt severe right lumbar pains, which moves down to his inguinal area after 6 hours. The pain is on and off with periods of no pain at all. His urine has turned bright red. His father had history of the same symptoms. D 69. Mr. Aniceto, 55 year old, has attacks of ankle and knee joint pains for over a year now which is often relieved by taking diclofenac and mefenamic acid. Recently, he sought consult because of pallor and easy fatigability. B 70. The Isleta family is known for having polycystic kidneys. The mother and 3 children have the disease. The mother is now on dialysis due to end stage kidney disease. B Column B a. b. c. d.

Glomerulus Tubules / Interstitium Urinary bladder Ureter

MCQ. Choose the best answer: 71. These are the clues to the diagnosis of obstructive syndrome, EXCEPT: a. Anuria b. Azotemia c. Proteinuria d. Urinary retention 72. A woman is diagnosed to e suffering from acute urethral syndrome. These are the expected features of this syndrome, EXCEPT: a. Dysuria b. Fever c. Bacteriuria but negative culture d. Frequency 73. A 50 year old, man, has been suffering from polycystic kidney disease. His creatinine now is very high and almost in the end stage. You expect his ultrasound of kidneys will show: a. Contracted size b. Dilated ureters c. Smooth borders d. Enlarged kidneys 74. An elderly man suddenly developed anuria. This condition may signify: a. Infection b. Dehydration c. Shock d. Obstruction 75. Dona Buding, 55 y/o, has stage IV ovarian malignancy. Now is admitted due to flank pains, fever, dysuria, and reduction in urine output. She has not improved even with the best antibiotics. An ultrasound is in order to get clue on: a. Status of renal function b. Obstruction c. Nature of the organism d. Presence of stone

76. Papillary necrosis is common in these conditions, EXCEPT: a. Analgesic nephropathy b. Diabetic mellitus c. Chronic pyelonephritis d. Chronic glomerulonephritis 77. These organisms are the likely cause of UTI in women presenting with dysuria, frequency, but with no growth on culture, EXCEPT: a. coli b. Chlamydia c. Neisseria gonorrhea d. Herpes simplex 78. Leukocyte casts are pathognomonic of this condition: a. Chronic glomerulonephritis b. Acute pyelonephritis c. Acute cystitis d. Interstitial nephritis 79. The best treatment option for a 20 year old woman presenting with frequency, hematuria, and hypogastric pain is: a. 7 day course of quinolone b. 3 gm amoxicillin single dose c. 7 days nitrofurantoin d. Parenteral ceftriaxone 80. Low dose suppressive antibiotic therapy is indicated in this type of kidney stone: a. Struvite b. Calcium oxalate c. Uric acid d. Cystine 81. The diagnosis of the syndrome of asymptomatic urine abnormalities is based on the following findings, EXCEPT: a. Hypertension b. Hematuria c. Sterile pyuria d. Mild proteinuria 82. These serologic findings are commonly (+) in lupus nephritis, EXCEPT: a. IgG b. Anti-ds DNA c. ANCA d. Anti – GBM 83. The characteristic feature of crescentic glomerulonephritis is: a. Proteinuria b. Renal failure over weeks to few months c. Edema d. Active urine sediments 84. Immunologic glomerular injury is found in this condition: a. DM nephropathy b. Primary focal segmental glomerulonephritis c. Hypertensive nephropathy d. Amyloid nephropathy 85. This condition is associated with membranous glomerulonephritis: a. Hepatitis B and C b. HIV c. Hodgkin’s lymphoma d. Heroin abuse

86. The leading cause of idiopathic nephritic syndrome in adults is: a. Minimal change disease b. Membranous GN c. Membranoproliferative GN d. IgA Nephropathy 87. This drug can cause minimal change glomerulonephritis: a. NSAID b. Allopurinol c. Warfarin d. Captopril 88. This condition should always be considered in the evaluation of elderly patient presenting with nephrotic syndrome: a. Malignancy b. Minimal change disease c. Rapidly progressive. d. Urinary tract infection 89. Microalbuminuria is a strong predictor for the development of this condition: a. Chronic renal failure b. Acute glomerulonephritis c. Diabetic glomerulosclerosis d. Chronic pyelonephritis 90. In nephrotic syndrome, patients have very low plasma albumin resulting to anasarca, which can be explained by: a. Decreased oncotic pressure b. Increased hydrostatic pressure c. Decreased plasma volume d. Hypotension 91. The best way to approach pre - renal failure is: a. Close observation b. Infusion of inotropic drugs c. Intravenous diuretics d. Prevention 92. In this particular stage of chronic renal failure, patient is still asymptomatic but when investigated will show that renal reserve is diminished sufficiently: a. Ischemic stage b. Uremia c. Nephrotic syndrome d. Azotemia 93. Uremic symptoms are caused mostly by this product of amino acid metabolism: a. Guanidine b. Creatinine c. Urea d. Lysine 94. This is a common finding in chronic renal failure: a. Contracted kidneys b. Normocalcemia c. Metabolic acidosis d. Clubbing of fingers

95. The maintenance phase of acute renal failure ordinarily lasts for how many days?

a. b. c. d.

3 – 5 days 7 – 14 days 18 – 28 days 30 – 40 days

96. Mr. Dimasira, 26 year old, was stabbed at the back while walking along EDSA. He lost a lot of blood and was hypotensive when rushed to the ER. The most likely consequence of this incident to his kidneys is: a. Chronic renal failure b. Acute intrinsic renal failure c. Pre – renal failure d. Post renal failure 97. A 27 year old, man, had history of rises of BP since age 20 which was ignored until a year ago, he started to complain of nape pains and BP was 180 / 110. His creatinine was 4 mg% with small kidneys on ultrasound. Which of these laboratory results from the serum is compatible with the patient’s condition? a. High phosphorus b. High calcium c. Normal potassium d. Low sodium 98. The most acceptable explanation for the oliguria of acute renal failure is: a. Rise in BP b. Reduced GFR c. Ureteral obstruction d. Vasoconstriction 99. The muddy brown urine is pathognomonic of which condition: a. Acute tubular necrosis b. Post renal failure c. Pre – renal failure d. Chronic renal disease 100. a. b. c. d.

The most common cause of pre – renal failure is: NSAID Hypercalcemia Volume depletion Glomerulonephritis

DERMA 101. A. B.

102. A. B. C. D.

103.

The mode of transmission of pediculosis pubis is Sexual contact C. Contact with infected animals Airborne D. Arthropod bite Characteristics of the itch mite include the following EXCEPT: It is an oval, ventrally flattened mite. It can live on beddings for 7 days. The male dies after copulation. The female lays 2-3 eggs per day.

D.

Childhood atopic dermatitis is characterized by the following EXCEPT Lesions are less exudative, drier and more papular Pruritus is a constant feature Lesions are lichenified plaques The face and scalp are more often involved

A.

The most frequent cause of urticaria and angioedema is /are Food C. Pollen

A. B. C.

104.

B.

105. A. B. C. D.

106. A. B.

107. A. B.

108. A. B.

109. A. B.

C. D.

110. A. B.

Drugs

D. Alcohol

Therapy for severe anaphylaxis include the following EXCEPT: Subcutaneous epinephrine with a 1:1000 dilution Nebulization with 5% metaproterenol IV hydrocortisone IV antibiotics Guttate psoriasis is usually preceded by Trauma C. Streptococcal pharyngitis Hepatitis D. Viral exanthema “Mother” patch or “herald” patch is characteristic of Atopic dermatitis C. Pityriasis rosea Psoriasis D. Pityriasis alba Lacy white network on the surface of papules of lichen papules is called Apple jelly sign C. Koebner phenomenon Wickham’s striae D. Woronoff sign The 3 zones of the target lesion of Erythema multiforme minor are Central pale area; a purpuric area; surrounded by a ring of erythema Central dusky purpura; an elevated edematous pale ring; a surrounding macular erythema A central pale area, a ring of erythema; a surrounding erythema Central dusky purpura; a ring of erythema; surrounding edematous pale ring The primary lesions of shingles are Papules C. Vesicles Crusts D. Pustules

111. This is a bullous type of drug eruption with involvement of the mucosal surfaces Steven-Johnson syndrome C. Exfoliative dermatitis A. Acute urticaria D. Fixed drug eruption B. 112. The following are cutaneous manifestations of Systemic Lupus Erythematosus EXCEPT: Leg ulcers C. Telangiectasia A. Calcinosis cutis D. Sclerosis B. 113. This is a febrile, rapidly evolving, generalized, desquamative infectious disease, in which the skin exfoliates in sheets. Steven-Johnson syndrome A. Toxis Shock syndrome B. Staphylococcal Scalded Skin Syndrome C. Toxic Epidermal Necrolysis D. 114. A. B. C. D.

The etiologic agent of Erysipelas is Staphylococcus aureus Pseudomonas Group A Beta-hemolytic Streptococcus Erysipelothrix rhusiopathiae

115. The fungi that usually cause only superficial infection of the skin include the following EXCEPT: Microsporum C. Trichophyton A. Candida D. Epidermophyton B.

116. A. B.

117. A. B.

118. A. B.

119. A. B.

120. A. B.

121. A. B.

Hansen’s disease with a single anesthetic patch is classified as Tuberculoid C. Borderline Indeterminate D. Lepromatous Organism implicated in acne vulgaris Staphylococcus aureus C. Streptococcus pyogenes Malazessia furfur D. Proprionibacterium acnes Pathogenesis of alopecia areata Abnormal cell-mediated immune factors Genetic

C. Emotional stress D. All of the above

The clinical hallmark of acne vulgaris is Papule C. Comedo Pustule D. Scar These viral diseases may be transmitted sexually EXCEPT: Herpes zoster C. Herpes simplex condyloma acuminate D. Molluscum contagiosum The following are deep types of folliculitis EXCEPT: Carbuncle C. Impetigo of Bockhart Cellulitis D. Abscess

122. This is a generalized disorder of connective tissue in which there is fibrous thickening of the skin with involvement of the internal organs. Systemic Lupus Erythematosus C. Hansen’s disease A. Systemic sclerosis D. Eosinophils fasciitis B. 123. A. B.

124. A. B.

125. A. B. C. D.

126. A. B. C. D.

127. A. B. C. D.

The hormone implicated in the pathogenesis of Androgenetic alopecia is: Testosterone C. Progesterone Growth hormone D. Estrogen The most frequent single nerve affected in herpes zoster is: Olfactory C. Auditory Facial D. Trigeminal The pathognomonic sign of Dermatomyositis is: Flat-topped violaceous papules over the knuckles Swollen pinkish violet eyelids with telangiectasis Pitting edema on the shoulder girdle, arms and neck Subcutaneous calcified nodules A superficial type of folliculitis is Carbuncle Cellulitis Sycosis barbae Abscess The most severe of the rickettsiae infection is caused by Rickettsiae tsutsugamushi Rickettsiae prowazecki Rickettsiae akari Rickettsiae rickettsii

128. A vesicular eruption of the palms and soles characterized by eczematous patches containing intraepidermal vesicles is Nummular eczema C. Housewife’s eczema A. Dermatophytid reaction Dyshidrotic eczema B. 129. This is the pathognomonic sign of psoriasis elicited by removing the scale revealing pinpoint bleeding points underneath it.

A. B. C. D.

CASE :

Koebner phenomenon Woronoff ring Auspitz sign Carpet tacks A 52 year old male consulted at the OPD due to appearance of an erythematous well-defined plaque with a sharply defined border and an atrophic center on the right elbow. The lesion first appeared 2 year ago. Skin prick testing revealed 100% anesthesia.

130. A. B. C. D.

What is the expected bacillary count if you take a tissue smear? 0 3+ 6+ 5+

131. Which test indicates the mean score of the number of organisms taken from different sites Skin punch biopsy A. Tissue smear B. Mitsuda reaction C. Histamine test D. 132. A. B. C.

D.

The following are true regarding Hansen’s disease EXCEPT The tuberculoid pole indicates poor resistance to the bacilli. Some families are genetically predisposed to developing the disease Majority of exposed individuals are transiently affected but few manifest the disease. It is transmitted through chronic contact with other persons with leprosy.

133. An inflammatory myositis without the skin changes of dermatomyositis is called polymyositis A. Gottron’s sign B. Myopathy C. Myalgia D. 134. This is a generalized disorder of connective tissue in which there is fibrous thickening of the skin with involvement of the internal organs CREST syndrome A. Systemic sclerosis B. SLE C. Eosinophilic fasciitis D. 135. A. B. C. D.

136. A. B. C. D.

137.

The hormone implicated in the pathogenesis of SLE is Testosterone growth hormone progesterone estrogen The most frequent dermatome affected in herpes zoster is sacral facial thoracic lumbar

This is a clinical feature of Staphylococcal Scalded Skin Syndrome

A.

B. C. D.

138. A. B. C. D.

139. A. B. C. D.

140. A. B. C. D.

141. A. B. C. D.

142. A. B. C. D.

143. A. B. C.

D.

144. A. B.

C.

D.

It is a febrile, rapidly evolving, generalized disease in which the skin exfoliates in sheets. Nikolsky sign is not elicited. Culture of the skin is positive for Staphyloccus aureus. The palms, soles and mucous membranes are often involved. The characteristic itch of scabies is intermittent continuous diurnal nocturnal The pathognomonic sign of Dermatomyositis is flat-topped violaceous papules over the knuckles swollen pinkish violet eyelids with telangiectasis pitting edema on the shoulder girdle, arms and neck subcutaneous calcified nodules The most frequent initial symptom of SLE is joint pain weight loss malar rash oral ulcers The basic abnormality in psoriasis is accelerated epidermopoiesis viral invasion abnormal reaction to drugs spongiosis The treatment of choice for scabies in infants and pregnant women is Sulfur lotion Ketoconazole Permethrin Corticosteroids Which is true regarding the management of SLE? Application of sunscreen is not important. There is no cure for SLE. Nonsteroidal anti-inflammatory drugs is enough for life-threatening manifestations. Complete remissions are often seen. Which is true regarding herpes simplex? Acyclovir is recommended for routine use in recurrent episodes. Prophylaxis with acyclovir is recommended for patients with recurrent herpes simplex before undergoing facial surgery. Treatment of sexual partners of patients with genital herpes is not necessary. None of the above

145. Alkali such as soaps, detergents and bleaches produce this type of dermatitis allergic contact dermatitis A. irritant contact dermatitis B. nummular eczema C. impetigo contagiosa D.

146. A.

Berloque dermatitis is usually seen on preauricular area and side of the neck

B. C. D.

V area of the chest scalp back

147. These skin diseases are highly infectious among household contacts EXCEPT scabies A. pediculosis B. impetigo contagiosa C. psoriasis D. 148. A. B. C. D.

Which is a scarring type of alopecia? Telogen effluvium Alopecia areata Androgenetic alopecia Discoid lupus erythematosus

149. Skin eruption characterized .by circular or oval “coinlike” patches over the trunk and extensor surfaces of the extremities Contact dermatitis A. Psoriasis B. Lichen simplex chronicus C. Nummular eczema D. 150. A. B. C. D.

151. A. B. C. D.

152. A. B. C. D.

153. A. B. C. D.

154. A. B. C. D.

The clinical hallmark of acne vulgaris is inflammatory papules pustules comedones cysts The characteristic lesion os Seborrheic dermatitis is greasy scales on the eyebrows, eyelids, glabella and nasolabial fold thick silvery-white scales over the scalp and trunk dry patches and crusted plaques over the cheeks salmon-colored oval patches with collarette scaling over the trunk following a “Christmas tree” pattern Which is true regarding verruca (wart)? It is viral in etiology. It is contagious in children only. It may be treated with griseofulvin. It is mainly found in patients with a lymphoma. The characteristic of ringworm fungi as compared with other fungi is their ability to digest and hydrolyze keratin high degree of contagiousness ability to invade the dermis sensitivity to penicillin The best treatment for atopic dermatitis includes psychoanalysis warm clothing a change of environment vigorous exercise

155. Treatment of acute contact dermatitis during the bullous, oozing stage should include topical anesthetics A.

B. C. D.

bland compresses and baths corticosteroid ointments systemic antibiotics

CARDIOLOGY

LEANDRO BONGOSIA, M.D.

156. CV disease is presently considered as the leading cause of death worldwide. Which of the following conditions is the most common? Congenital Heart Disease C. Coronary Heart Disease A. Rheumatic Heart Disease D. Cardiomyopathies B. 157. A. B.

Extracardiac obstructive shock is exemplified by: Acute myocardial infarction C. Cardiac Tamponade Arrhythmia D. Ventricular aneurysm

158. Mrs. Santiago went into fasting for 7 days and 7 nights without food and water. She eventually went into shock. Her shock is classified as: Hypovolemic C. Distributive A. Extracardiac obstructive D. Cardiogenic B. 159. One of the following considerations is not part of a complete cardiac diagnosis based on the NYHA: Underlying etiology C. Hemodynamic abnormalities A. Anatomic abnormalities D. Functional and therapeutic classification B. 160. to?

The anatomic abnormality of a patient with coronary artery disease is due A. B.

Stenotic aortic valve Stenotic ascending aorta

C. Atherosclerotic narrowing of the heart D. Shunt anomaly

161. Dizziness and syncope are common in which of the following pathophysiologic mechanisms of CV disease. Myocardial infarction A. Dysrhythmias B. Systolic and diastolic myocardial dysfunction C. Valvular blood flow obstruction D. 162. Among the non invasive procedures available for CV examination, the one which gives adequate information on valvular structures, chamber size and function is…. A. Chest x-ray C. Nuclear imaging B. 2-D Echocardiogram D. MRI 163.

The frequency of acute myocardial infarction is highest in: A. Early afternoon C. Midnight B. Early evening D. The morning within a few hours of awakening

164. A 60 years old male executive was brought to the ER with history of progressive moderate to severe chest pain. A known smoker and a Type II Diabetic. PE, BP=140/90, CR=100/min., RR=28/min., Heart=grade 3/6 holosystolic murmur heard best at the apex radiating to the back, Lungs= occasional bibasal crackles, Extremities=no edema. ECG showed ST elevation in V1-V2, V3 CK-MB enzyme was 3 times elevated than normal. Your most likely diagnosis is? Unstable angina C. Acute lateral wall MI A. Acute anteroseptal MI D. Acute inferior wall MI B. 165. The above was given initial standard medicines and then four hours later he developed acute cardiopulmonary distress, CR=110/min., RR=30/min., BP=110/70 unrelieved by O2 inhalation. What complication is he now having?

A. B.

166. A. B.

167. A. B. C. D.

Cardiogenic shock Acute pulmonary embolism

C. Acute pulmonary edema D. Aortic dissection

What is the normal PR interval on ECG in adults? Up to 0.02 sec. C. Up to 0.04 sec. Up to 0.20 sec D. Up to 0.4 sec. The early initial pathologic changes seen in atherosclerosis. Formation of “foam cells” Formation of fibrous cap and lipid core Lipoprotein accumulation and “fatty streak” formation Adhesion and penetration of leukocytes into the intima

168. Atherosclerotic plaque is considered vulnerable if it has one of the following features: Abundant smooth muscles cells C. Small lipid core A. B.

Scanty macrophages

A.

Management of deep vein thrombosis include the following EXCEPT? Heparin C. Ibuprofen Coumadin D. Streptokinase

169. B.

D. Thin fibrous cap

170. The definitive diagnostic procedure prior to a bypass procedure of the peripheral vasculature is: Duplex scan C. CT scan A. Magnetic resonance imaging D. Arteriography B. 171.

Secondary prevention programs includes…… Identification and management of risk factors to prevent CHD in high A. risk asymptomatic patients Preventive measure programs to prevent recurrent events in patients B. with established disease Preventive and management program in patients with existing C. complications Surgical options in patients with established disease D.

172. A 60-year old diabetic patient consulted because of failure symptoms. EKG revealed old anterior wall MI, urinalysis-2+ albumin. The drug of choice in the treatment of hypertension for this patient is: Betablocker C. ACE inhibitor A. Calcium antagonist D. Diuretic B. 173. Accelerated atherosclerosis in diabetic patients is due to metabolic causes. Which of the following lipid abnormalities is usually noted in diabetic patients?  LDL  triglyceride  HDL C.  LDL  HDL  triglyceride A.  LDL  HDL  triglyceride D.  LDL  HDL  triglyceride B. 174. A 40 year old male has a BP of 170/105 on routine examination. This BP based on the JNC 7 classification is considered as stage: I C. III A. II D. IV B. 175. A. B.

The antihypertensive drug which causes reduction in pulse rate is: Spironolactone C. Metoprolol Captopril D. Nifedipine

176. A. B.

Risk factor which is considered non modifiable: LDL cholesterol C. Hypertension Gender D. Type 2 Diabetes Mellitus

177. Based on the Keith-Wagener classification of the fundus in hypertension, a grade 4 retinopathy changes include: Arteriovenous nicking C. Arteriolar narrowing A. Hemorrhages and exudates D. Papilledema B. 178. In acute myocardial infarction, Killip II clinical classification include the following EXCEPT: S3 gallop C. Tachypnea A. Orthopnea D. Diaphoresis, peripheral cyanosis and mental confusion B. 179. The hemodynamic hallmark of mitral stenosis secondary to rheumatic disease is characterized by elevation of : left atrio-ventricular pressure gradient A. left atrial pressure B. C. pulmonary arterial pressure D. pulmonary capillary pressure 180. Which of the following valvular conditions is associated with an Austin Flint murmur? Aortic stenosis C. Mitral regurgitation A. Pulmonic stenosis D. Aortic regurgitation B. 181. A 30 year old office worker was admitted at the medical ICU because of sudden onset of substernal chest pains, non effort related, lasting for about 30 mins. This was associated with cold clammy perspiration. His 12 lead ECG showed Q waves with ST elevation on leads II, III, and AVF. This patient has: Anteroseptal wall MI C. Posterior wall MI A. Anterolateral wall MI D. Inferior wall MI B. 182. Above patient complained of persistent chest pains. In such cases, the following drugs can be used in controlling the pain associated with myocardial infarction EXCEPT: Morphine C. Meperidine HCI A. Nifedipine D. Nitroglycerin B. 183. The most common complication of myocardial infarction within 24 hours after the event is: Ventricular fibrillation C. Dressler’s syndrome A. Myocardial rupture D. Left ventricular aneurysm B. 184. Which one of the following conditions could result to systolic hypertension with wide pulse pressure due to increased stroke volume: Thyrotoxicosis C. Patent ductus artenosus A. Aortic regurgitation D. Pheochromocytoma B. 185. A. B.

The following are adverse metabolic effects of thiazides EXCEPT: Hypokalemia C. Hypercholesterolemia Hypoglycemia D. Hyperuricemia

186. A 16-year old male is diagnosed to have mitral stenosis due to rheumatic heart disease. Which of the following physical exam findings is NOT compatible with his case? Apical diastolic rumble C. Left ventricular heave A. Engorged neck veins D. Opening snap B.

187. A 30-year old male chronic IV drug user presents with high grade fever, dyspnea, chest pain, tachycardia without any appreciable murmur. A diagnosis of infective endocarditis is made and the valve most probably affected is the: Mitral C. Tricuspid A. aortic D. Pulmonic B. 188. A 20-year old laundry woman has mitral regurgitation due to RHD. Lately she noticed shortness of breath and fatigue while doing her usual laundry work. What is her present functional classification? Class I C. Class III A. Class II D. Class IV B. 189. A patient with known CAD hypertension is assessed to be in congestive heart failure NYHA III-C class. PE reveals BP=100/70, HR=60/min.RR=22, engorged neck veins, bibasal crackles & S3 gallop. Which of the following medications is least likely to improve the patient’s hemodynamics? Metoprolol C. Furosemide A. Captopril D. Spironolactone B. 190. A 22-year old female is a diagnosed case of RHD mitral stenosis & mitral regurgitation. She now presents with fever and petechial lesions of the skin and conjunctivae. A definite diagnosis of infective endocarditis can be made by: Echocardiography C. Electrocardiography A. Blood culture D. Chest x-ray B. 191. A 45-year old male was admitted because of acute myocardial infarction. On his 2nd day at ICU, he developed a grade 3/6 systolic murmur heard best at the apex. This is most probably due to: Mitral stenosis C. Tricuspid stenosis A. Papillary muscle dysfunction D. Aortic regurgitation B. 192. A 40 year old male was seen at the clinic for the 1 ST time with a BP 150/90. He should be started immediately with antihypertensive drugs if: He is a smoker C. There is evidence of target organ damage A. He is overweight D. He is fund of eating salty foods B. 193. A 48-year old male smoker consulted the clinic due to sudden onset of severe substernal chest pain, dyspnea with cold clammy perspiration. BP 100/70, HR 112/min., (+) crackles all over both lung fields, (+) S3 gallop (-) murmur. ECG showed T wave inversions in V1 – V6, (+) Troponin T. Your likely diagnosis is: Unstable angina C. Non-ST elevation MI (NSTEMI) A. ST elevation MI (STEMI) D. Acute aortic dissection B. 194. A 60-year old male hypertensive consulted the clinic because of effort related dyspnea and easy fatigability. BP 180/90, HR 90/min., (-) neck vein engorgement, clear breath sound. Apex beat 6th ICS left MCL with grade 3/6 systolic murmur heard best at the apex, (-) edema. The murmur is due to: Mitral stenosis C. Mitral regurgitation A. Aortic stenosis D. Aortic regurgitation B. 195. A. B.

196. A. B.

Which of the following is a late finding in left heart failure? Soft S1 C. Loud S2 Pulmonary crackles D. Elevated bilirubin Which of the following conditions could result to high output heart failure? Hypertension C. Thyrotoxicosis Valvular heart disease D. Dilated cardiomyopathy

197. A 60 year old male presented with cold clammy perspiration and severe chest pains. He is known diabetic and a 20 pack-year smoker. At the emergency room, patient is in severe repiratory distress with crackles all over both lung fields. Which of the following diuretics is best given in this case ? Hydrochlorothiazide C. Bumetanide A. Furosemide D. Spironolactone B. 198. The pathophysiologic mechanism by which smoking may promote atherosclerosis include the following factors, EXCEPT: Decrease oxidation of LDL and increase level of HDL cholesterol A. Impairment in endothelial function B. Increase inflammatory markers C. Platelet aggregation D. 199. A 16 year old high school basketball player noticed that he now tires easily compared to his team mates.Midway during one practice session, he was already out of breath. The school physician was consulted and noted the presence of right ventricular lift, fixed wide splitting of S2 and a grade 3/6 systolic murmur at the 2nd left intercostal space. The diagnosis is: Rheumatic heart disease (RHD), mitral stenosis A. RHD, mitral regurgitation B. Congenital heart disease (CHD), atrial septal defect C. CHD, ventricular septal defect D. 200. A. B.

High levels found to be good for the heart. Chylomicrons C. HDL LDL D. Triglycerides

201. Among the drugs with lipid lowering effects the one which have shown highly effective in lowering total cholesterol, LDL and triglycerides but may induce myopathy and increase liver enzymes is…. Bile acid requestrants C. Statins (HMG-COA reductase inhibitors) A. Nicotinic acids D. Fibric acids B. 202. Sudden arterial occlusion may present with the following symptoms and signs distal to the site of occlusion, EXCEPT: Hyperemia C. paresthesia A. Pain and numbness D. Pallor & absence of pulse B. 203. Intermittent claudication involving the legs but sparing the thigh suggest obstruction of what artery: Popliteal C. Femoral A. Dorsalis pedis D. Radial B. 204. A 60-year old male was admitted because of a non-healing leg ulcer located on the proximal end of the left leg near the lateral malleolus associated with pitting edema and hyperpigmented skin. There is no tenderness nor exudates. Her glucose tolerance test is normal. She probably has: Neuropathy C. Chronic venous insufficiency A. Arterial occlusion D. Hypoalbuminemia B. 205. A. B.

The LDL target for patients with established CAD is less than: 100 mg/dl C. 160 mg/dl 130 mg/dl D. 200 mg/dl

CLINICAL IMMUNOLOGY CASE :

A 30-year-old female presents with fever, diffuse inflammatory arthritis, weight loss, malar rash, discoid rash, alopecia, mouth ulcers, and pleuritic chest pain. The following are the laboratory findings: Anemia Thrombocytopenia Urinalysis: 8 RBC, 10WBC, +++ protein, Glomerolonephritis ANA: positive DsDNA: + C3 and C4: both low Anti Smith: + ESR: is elevated at 67 (NV: 0-15)

206. A. B.

207. A. B.

208. A. B.

What is the most likely diagnosis? Osteoarthritis C. Systemic lupus Rheumatoid arthritis D. Post strep glomerolonephritis Which of the following is the most sensitive test for lupus? DsDNA C. Urinalysis ESR D. ANA Which of the following is/are the most specific test/tests in SLE? ANA C. C3 and C4 DsDNA and Anti-Smith D. ESR

209. In case of a suspected lupus flare or exacerbation, which of the following laboratory could be repeated to follow the disease activity of SLE? ESR, DsDNA, C3 and C4 C. Anti-Smith A. ANA D. Joint x-ray B. 210. A. B.

211. A. B.

Which of the following are manifestations of SLE except Cerebritis C. Joint erosion on x-ray Pericarditis D. Discoid rash The most important laboratory test to diagnose drug induce lupus is ESR C. Urinalysis ANA D. Anti-histone antibody

212. What is the best maintenance treatment for the constitutional symptoms and skin rash of SLE? Hydroxychloroquine (Plaquenil) C. Prednisone A. Non-steroidal (NSAID) D. Cyclophosphamide B. 213. A 26 year-old-female complain of fatigue, malaise and weight loss. Physical Examination shows patchy alopecia, malar rash, and arthritis of hands and feet. You suspects SLE and you are allowed to take only one test. What is the most appropriate screening test for this patient? ESR D. Anti-DsDNA and anti-Smith A. ANA E. C3 and C4 B. LE-prep C.

CASE :

A 28 year-old-female presents with fatigue, muscle pain, rashes over the face and knuckle. She has difficulty combing her hair, reaching an over her head and arising from bed or chair. Physical examination lilac colored rash on the eyelids, erythematous rash on her grade 3/5 motor power of the proximal muscles and the rests normal.

object shows knuckles, of PE are 214. A. B.

215. A. B.

216. A. B.

217. A. B.

What is the most likely diagnosis? SLE C. Dermatomyosisis Rheumatoid arthritis D. Cardiovascular accidents What is the best test to be requested? Urinalysis C. Creatinine phosphokinase (CPK or CK) ANA D. CT scan What is the best test for the definitive diagnosis of her condition? Skin biopsy C. MRI of the brain Muscle biopsy D. MRI of the spine The typical rash in the face and knuckles is/are called: Gottrons rash and heliotrope rash C. Malar rash Heliotrope rash D. Discoid rash

CASE :

A 42-year-old female comes in because of extreme dryness of the eyes due to, mouth and tongue (keratoconjunctivitis sicca). She also has fatigue, one-hour morning stiffness and arthritis of the fingers. On physical examination, both parotid glands are swollen but not tender. ESR is elevated at 30mmhg. 218. A. B.

219. A. B.

What is the most likely diagnosis? Sjogren's syndrome Systemic lupus

C. Rheumatoid arthritis D. Polymyalgia rheumatica

The most likely positive antibody is/are: Anti-Jo 1 C. Anti- RNP Anti-Scl D. Anti Ro/ anti La (anti SS-A/anti SS-B)

CASE :

A 70-year-old female came to your clinic because of sudden blurring of both eyes. 2 months ago she complained of fatigue, 2-hour morning stiffness of the hip, back neck and shoulder. 1 week ago she started having jaw claudication, and temporal headache. On examination there is tenderness of the scalp over the temporal area. 220. A. B.

221. A. B.

What is the most likely diagnosis Cerebrovascular accident (CVA) Carotid artery stenosis

C. Polymyalgia rheumatica D. Giant cell arteritis

Which of the following is the most appropriate management? Heparin C. High dose prednisone Emergency operation of the carotid artery D. Observation

222. Which of the following procedures should be requested to confirm your diagnosis? Temporal artery biopsy C. Echocardiogram A. MRI of the brain D. Direct fundoscopy B.

CASE :

A 17-year-old female is admitted because of skin rash and severe abdominal pain. A week ago she had flu like symptoms manifested by fever, non-productive cough, arthralgia and malaise. Physical examination reveals diffuse palpable purpura. PE shows diffuse palpable purpura in the lower extremities and swollen knees and ankles. Urinalysis shows protenuria and hematuria, Creatinine is normal. ANA is negative. 223. A. B. C. D.

224. A. B.

Your most likely diagnosis is: Systemic lupus erythematosus Henoch-Schonlein purpura Post infectious arthritis due to viral infection Post strep Glomerulonephritis Kidney biopsy would most likely show: Glomerolosclerosis Diffuse crescentic glomerulonephtitis

C. IgA nephropathy D. Normal biopsy

225. A 30-year-old female is admitted because of dizziness, headache and claudication of the upper extremities. Left upper blood pressure is absent. Right upper blood pressure is 60/10. Dorsalis pedis pulses were normal and blood pressure of the lower extremity is 130/70. ESR is 75mmhg. The most likely diagnosis in this case is? Temporal arteritis C. Takayasus arteritis A. Coarctation of the aorta D. Dissecting thoracic aneurysm B. 226. A 24-year-old male comes in because of recurrent aphtous and genital ulcerations for 1 year. There is swelling of both ankles. Pathergy test is positive. The most likely impression is? Reiter’s syndrome C. Gonococcal arthritis A. Behcet’s disease D. Herpes simplex B. CASE :

A 55-year-old female presents to your clinic because of diffuse skin thickening, raynauds, esophageal dismotility ant telangiectasia. Blood pressure is 180/112. Her physical examination reveals skin thickening of all extremities and face. There are telangiectasias of the fingers. Creatinine is elevated. Barium swallows shows esophageal dysmotility. 227. A. B. C. D.

CASE :

Which of the following is the most likely diagnosis? Sjogren’s syndrome Systemic sclerosis (scleroderma) Primary Raynaud’s syndrome Sarcoidosis

had “sore

An 18-year-old male has been complaining of low back pain for 9 months. He has been experiencing 2 hours morning stiffness fatigue. He eyes” 3 months ago that was diagnosed as uveitis

with

On physical examination, there is tenderness of the lumbar and bilateral sacroiliac area. ESR is 45mm/hg. X-rays shows “bamboo spine” bilateral sclerosis and erosion of both sacro-iliac joints.

228. A. B.

229. A. B.

What is the most likely diagnosis Polymyalgia rheumatica C. Wegener’s Granulomatosis Polyarteritis nodosa D. Ankylosing spondylitis The Human Leukocyte antigen (HLA) associated in this disorder is? MHC 4 C. HLA-B27 HLA-DR 4 D. HLA DP, DQ, DR

CASE :

A 45-year-old female comes in to your clinic for second opinion of asthma diagnosed 6 months ago. Physical examination is remarkable for of ankle and wrists. There are diffuse bilateral dry rales. WBC: has significant eosinophilia Chest x-ray: diffuse interstitial infiltrate

arthritis

230. A. B.

The disease is most likely? Adult onset asthma Sarcoidosis

C. Churg Strauss syndrome D. Polyarteritis nodosa

RHEUMATOLOGY 231. A. B. C. D.

232. A. B. C. D.

233. A. B.

234. A. B. C. D.

235. A. B.

The pain in osteoarthritis arise from the following structure EXCEPT: Articular cartilage erosion Stretching of joint capsule Stretching of periosteum covering osteophytes Synovitis The following are characteristics of gouty arthritis EXCEPT: Acute in onset Plasma uric acid level may not be elevated in all cases Uric acid lowering agent is necessary during the acute attack Usually monoarticular The crystals found in the synovial fluid of pseudogout. Calcium oxalate C. Cholesterol Calcium pyrophosphate dehydrate D. Monosodium urate Acute gout pathogenesis results from: Dehydration Deposition of monosodium urate crystals in joints Ethanol ingestion Interaction of urate crystals and PMN The most common route by which bacteria enter the joint: Contiguous site of infection C. Hematogenous Direct inoculation D. Trauma

236. A rubifacient drug whose mechanism of action is to deplete substance-P in the local nerve ending. Capsaicin cream C. NSAID cream A. Mentholated cream D. Tramadol B. 237. A. B.

Not a tender point site. Gluteal Low cervical

C. Scapular D. Trapezius

238. The most common site of osteoarthritis in the elderly in developed countries: Ankle C. Hand A. Knee D. Hips B. 239. A. B.

240. A. B.

The force applied to a tender pint of fibromyalgia. 40 gms C. 4,000 gms 400 gms D. 40,000 gms One of the following will manifest with chronic monoarticular arthritis. Crystal induced C. Mycobacterium tuberculosis Fracture D. Staphylococcus aureus

241. A 67-year-old male is admitted under your service because of myocardial infarction. On the 4th hospital day, he develops acute swelling of left knee and right ankle. Aspiration was done and it shows slightly positive birefringent crystal compatible with calcium pyrophosphate. What is your diagnosis? Gout c. Septic arthritis A. Pseudogout or CPPD d. Reactive arthritis B. 242. A 22-year-old intravenous drug abuser with a past medical history of rheumatic Fever comes to you because of fever and acute right wrist pain and swelling. He has Osler’s node and dark tiny discolorations under his nail beds. What are you going to do at this time? Start IM benzathine Penicillin A. Admit and start corticosteroid B. Request Echocardiogram, joint aspiration and start appropriate antibiotic C. Do not do anything. It is waste of time for this person D. 243. A 52-year-old female presents with progressive, Bilateral, symmetrical diffuse arthitis for 5 years. She has 3 hours morning stiffness. Physical examination is remarkable for mild to moderate swelling of PIP, MCP, wrists, elbows, knees, ankles and toes. ESR is elevated at 68. X-ray of the hands shows periarticular osteopenia and joint erosions. What is the most likely diagnosis? Osteoarthritis C. SLE A. Rheumatoid arthritis D. Fibromyalgia B. 244. Rheumatoid factor (RF) in this patient is negative. What is true statement below regarding Rheumatoid factor? The diagnosis is unlikely rheumatoid arthritis if RF is negative A. Rheumatoid factor is specific for rheumatoid arthritis B. RF in low titer can be present in normal individual C. It is always present in Rheumatoid arthritis D. 245. A. B. C. D.

246. A. B. C. D.

The following are manifestations of rheumatoid arthritis except. Swan neck deformity Boutonniere deformity Joint erosions and periarticular osteopenia Swelling and erosion of distal interphalangeal joint (DIP) The following are true of rheumatoid arthritis except. it is more common in female it has extraarticular manifestations (eyes, lungs, vasculitis) It may cause anemia of chronic disease In case of exacerbation ESR and RF should always be repeated because RF correlates well with disease activity.

247. The above patient did not respond to NSAID and low dose steroid for 3 months. What is the best thing to do at this time? add methotrexate A. change to another NSAID or increase the dose of corticosteroid B. Refer to Rheumatologist C. Leave her alone. This is a hopeless case. D. HEMA : DR. ENRICO TUY 248. All of the following are normally found in the peripheral blood of an adult EXCEPT: Lymphocytes A. Platelets B. Nucleated red cells C. Monocytes D.

249. A. B.

250. A. B.

251. A. B. C. D.

252. A. B.

253. A. B.

254. A. B.

The most appropriate site for bone marrow studies in adult is Sternum C. Anterior superior iliac crest Spinal process D. Posterior superior iliac crest The major hematopoietic organ in the fetal stage of development is the Bone marrow C. Spleen Liver D. Yolk sac The general signs and symptoms of anemia can be attributed to Reduction in oxygen carrying capacity of the blood Decrease plasma volume Increase blood viscosity Decrease in the affinity of hemoglobin for oxygen Which of the following is NOT caused by a structural membrane defect? Spherocyte C. Siderocyte Target cell D. Acanthocyte A patient with anemia whose mean cell volume is 120 fL would have a Macrocytic anemia C. Normocytic normochromic Microcytic anemia D. Hypochromic anemia A patient with severe anemia and a reticulocyte index of 4 has normal functioning marrow C. An adequate supply of iron Chronic renal disease D. A normal oxygen saturation

255. The first blood cell lineage to decrease after marrow injury with drugs, radiation, or infection is the: Neutrophils C. Red cells A. Platelets D. Lymphocytes B. 256. A. B.

257. A. B.

Which of the following conditions would show oval macrocytes? Thalassemia syndromes C. Anemia of chronic renal disease Megaloblastic anemia D. Myelodysplastic syndrome The primary factor that regulates erythropoietic activity is The kidney C. Oxygen Erythropoietin D. Bone marrow

258. Bone marrow biopsy is better over bone marrow aspiration in the evaluation of Marrow iron stores A. Evaluation of morphologic changes in the hematopoietic cells B. Infiltrative processes such as tumors, granulomas, and fibrosis C. Differential enumeration of the marrow cells D. 259. A. B. C. D.

Pancytopenia with an “empty” marrow is seen in Aplastic anemia Myelodysplastic syndrome Paroxysmal nocturnal hemoglobinuria Acute Leukemia

260. All of the following complications are associated with pancytopenia resulting from bone marrow failure EXCEPT: Sepsis C. Bleeding A. Signs and symptoms of anemia D. Thrombosis B. 261. A. B.

Which type of cell is most prominently seen in idiopathic myelofibrosis? Acanthocyte C. Schistocytes Burr cell D. Teardrop cell

262. A. B. C. D.

263. A. B.

Most cases of acquired aplastic anemia is due to: Direct injury to the pluripotent stem cell Destruction of the marrow microenvironment Deficiency in hematopoietic growth factors Immune-mediated injury to the hematopoietic stem cell The most common infection associated with aplastic anemia is Viral hepatitis C. Infectious mononucleosis Typhoid fever D. Parvovirus type B19

264. The treatment of choice for patients with aplastic anemia who lack a suitable marrow donor is: Immunosuppression with antithymocyte globulin (ATG) plus cyclosporine A. Androgen therapy B. Use of hematopoietic growth factors, G-CSF, GM-CSF, or IL-3 C. Splenectomy D. 265. All of the following clinical conditions are associated with decreased levels of erythropoieitin EXCEPT: Anemia of renal disease C. Polycythemia vera A. Autoimmune hemolytic anemia D. Anemia of hypothyroidism B. 266. Which of the following anemias is most likely to respond to the administration of erythropoietin? Iron deficiency anemia C. Pure red cell aplasia A. Pernicious anemia D. Anemia of renal disease B. 267. A. B. C. D.

Anemia in hypothyroidism is due to Accelerated red cell destruction Poor iron utilization Decrease O2 demand Inhibition of red cell production by cytokines

268. A 23-year-old woman was found to have severe iron deficiency. Her physician started her on oral ferrous sulfate, at 325 mg TID. Her hemoglobin 1 month later was unchanged. Fecalysis showed brown stools negative for occult blood. Which of the following reasons is the most likely explanation for the patient’s poor response to treatment? Malabsorption of iron A. Poor compliance B. Continued intermittent blood loss C. Poor release of iron from the preparation D. 269. A. B.

Dietary iron is predominantly absorbed in the: Stomach C. Proximal jejunum Duodenum D. Terminal ileum

270. The microcytic hypochromic anemias represent a related group of disorders with A quantitative defect in hemoglobin synthesis A. Decreased erythropoietin synthesis B. Defective nuclear maturation C. Cell membrane defect D. 271. A 58-year-old woman is found to be mildly anemic, presenting with a hemoglobin level of 10 g/dl and ferritin 9 ng/dl (normal = 10-200 ng/dl). Of the following courses of management, which one is best suited to this patient? Encourage a diet rich in iron A. Prescribe oral ferrous sulfate for 6 months B. Order bone marrow studies to identify iron deficiency C. Order stool analysis to detect the presence of occult blood. D.

272. A low to absent bone marrow iron stores is found in which of the following hypochromic, microcytic anemias? Anemia caused by lead intoxication C. Iron deficiency anemia A. Thalassemia D. Anemia associated with SLE B. 273. A. B. C. D.

The endothelial cell contributes to the initiation of thrombosis by: Releasing nitric oxide (NO) Synthesis and release of von Willebrand factor Producing prostacyclin Releasing t-PA

274. Acute disseminated intravascular coagulopathy (DIC) is characterized by all of the following laboratory abnormalities EXCEPT: Prolong prothrombin time and partial thromboplastin time A. Decrease fibrinogen level B. A normal platelet count C. Prolong thrombin time D. 275. A. B. C.

D.

276. A. B. C. D.

277. A. B. C. D.

A primary hypercoagulable state should be suspected in patient who is: A heavy smoker and developed coronary artery thrombosis Hypertensive and developed cerebral artery thrombosis Apparently healthy, active, and young man but developed deep vein thrombosis 50-year old woman who has heart failure and has pulmonary embolism Which of the coagulation factors has potential anticoagulant effect? Fibrinogen Prothrombin Factor VIII von Willebrand factor Heparin produces its anticoagulant effect by: Inhibiting factor Va and VIIIa Enhances inactivation of thrombin and factor Xa Interfering with vitamin K metabolism Binding with thrombomodulin

INFECTIOUS DISEASE : DR. PAMELA ROSE MATTI Fever and FUO 278. A. B. C. D.

Which of the following substances produce fever? pyrogens cytokines endotoxin exotoxin

279. A patient comes to you because of fever of unknown origin of 1 month duration. He has normal physical findings except for the fever. Several routine work-ups have already been done and results were normal. He was given Ciprofloxacin 500 mg 1 tablet twice a day for 1 week, still no lysis of fever. He then consults you, which of the following is your next step in patient care? Repeat all laboratory evaluations A. Will not not treat empirically with another antibiotic regimen since B. patient is stable Will do the Naprosyn test C. Give glucocorticoids D. Pathogenesis and Host Defense 280. Which of the following toxins has its mechanism of action in increasing cyclic adenosine monophosphate (cAMP)?

A. B. C. D.

281. A. B. C. D.

Shiga toxin Tetanospasmin SPE-A Cholera toxin Splenectomy predisposes to infection with which organsims? Staphylococcus, Neisseria, Haemophilus S. pneumoniae, Neisseria, Haemophilus Mycobacteria, Brucella spp., L. monocytogenes Mycobacteria, Neisseria, Haemophilus

282. A major mechanism of colonization exemplified by Staphylococcus epidermidis especially on intravascular catheter tips: Specific attachment to cellular surfaces through teichoic acids A. Adherence by adhesins B. Biofilm formation C. Antigenic variation D.

Antimicrobial Therapy 283. A. B. C. D.

284. A. B. C. D.

285. A. B. C. D.

The following antibiotic has a concentration-dependent activity: Penicillin Azithromycin Ciprofloxacin Clindamycin Which of the following are categories of beta-lactam antibiotics? Penicillins, quinolones, aminoglycosides, carbapenems Cephalosporins, quinolones, monobactams, carbapenems Monobactams, carbapenems, quinolones, aminoglycosides Penicillins, cephalosporins, monobactams, carbapenems Which of the following antibiotics target the 30S ribosomal subunits? Amikacin Chloramphenicol Erythromycin Clindamycin

Immunization 286. A. B. C. D.

Which vaccinations are important that should be given to elderly patients? MMR, Influenza virus Pneumococcal, Influenza virus Pneumococcal, Td Influenza virus, Td

287. Passive immunity in the form of antitoxins is used for the treatment and prevention of which bacterial diseases? Pneumococcal, Meningococcal, Typhoid fever A. Tetanus, Botulism, Diphtheria B. Tetanus, Botulism, Rabies C. Tetanus, Typhoid fever, Diphtheria D. 288. A. B. C. D.

Which of the following vaccines are contraindicated to pregnant women? Tetanus, Diphtheria Pneumococcal, Influenza MMR, Varicella Rabies, Hepatitis B

Sepsis 289. A 70-year-old man presents to the ER with a 2-day history of fever, chills, cough, and R-sided pleuritic chest pain. On the day of admission, the patient’s family noted that he was more lethargic and dizzy and was falling frequently. The patient’s vital signs are: T = 38.6°C; HR = 120 bpm; RR = 30 breaths/min; BP = 70/35 mm Hg; and O2 saturation as measured by pulse oximetry = 80% without oxygen supplementation. CXR shows a right lower lobe infiltrate. This patient’s condition can best be defined as which of the following? Multi-organ dysfunction syndrome (MODS) A. Systemic Inflammatory Response B. Septic shock C. Severe sepsis D. 290. A. B. C. D.

291. A. B. C. D.

During the inflammatory response of sepsis, which of the following occur? red blood cells release proinflammatory cytokines. TNF- attracts lymphocytes to the infection site. endothelial cellular injury occurs at the infection site. cytokines inhibit the coagulation cascade. The following is considered a mediator of septic shock: NO Vasopressin TNF IL-6

Gram positive cocci 292. Which of the following serves as a reservoir for community or hospitalacquired methicillin-resistant S.aureus? Eyes A. Ears B. Nose C. Throat D. 293. A 33 year old woman complains of fever, headache, muscle aches, nausea, and vomiting which began early this AM. 5 days ago she underwent a right inguinal hernia repair and had been recovering well until this morning. On exam she appears ill, BP 80/60, PR 120, RR 22, T 39.2. There is diffuse erythema of the skin, the conjunctivae are injected. What is your diagnosis? Cellulitis A. Non-menstrual TSS B. Necrotizing fasciitis C. Gas gangrene D. 294. The second most common cause of UTIs in sexually active younger women: Staphylococcus saprophyticus A. Staphylococcus aureus B. Staphylococcus epidermidis C. Enterococci D.

295. A 58 year old man comes to the physician because of extreme fatigue and malaise for 3 weeks. He has felt well except for a toothache 5 weeks ago treated with a root canal procedure. He has a history of a cardiac murmur first noted at the age of 19 years. His temperature is 37.8 o C , BP = 120/80 mmHg, and pulse is 110/min. The lungs are clear to auscultation. Cardiac examination shows a grade 2/6 systolic ejection murmur heard best at the right second intercostal space as well as an S4 and an ejection click. Laboratory studies shows: Hgb 9.3 g/dL WBC 10,000/mm3 Segmenters 90% Bands 10% ESR 90 mm/h Urine blood positive Blood cultures are obtained. Which organism is most likely to cause the above condition? Streptocococcus viridans A. Staphylococcus aureus B. Streptococcus pneumoniae C. Enterococcus spp. D. 296. A 59 year old male complains of fever and cough, with “rust” colored sputum. A CXR shows a dense infiltration of the left lower lobe and a left pleural effusion. Gram stain showed multiple PMNs and encapsulated gram-positive cocci in pairs and short chains. The causative agent of the patient’s pneumonia is: Legionella pneumophila A. Staphylococcus aureus B. Klebsiella pneumoniae C. Streptococcus pneumonia D. 297. A. B. C. D.

The causative agent of bacteremia in the presence of colon cancer: Streptococcus pyogenes Streptococcus pneumoniae Streptococcus bovis Streptococcus agalactiae

298. A 72 year old woman with an indwelling urinary catheter has a UTI and bacteremia. Gram positive cocci are isolated from the urine and blood cultures. Enterococcus faecalis A. Escherichia coli B. Proteus mirabilis C. Enterobacter aerogenes D. Gram negative cocci 299. A patient presents with fever, stiff neck, vomiting. Petechial lesions are present on the arms and chest. A Gram stain of CSF reveals the presence of bean-shaped, gram negative diplococci. What causes her meningitis? Streptococcus pneumoniae A. Haemophilus influenzae b B. Neisseria meningitides C. Listeria monocytogenes D. 300. A. B. C. D.

The antibiotic of choice for the patient in No. 28 is: Ceftriaxone Penicillin G Chloramphenicol Rifampicin

301. Chemoprophylaxis for meningococcal disease is indicated in which of the following? shaking hands with contact of index patient who have completed A. prophylaxis patients treated with ceftriaxone or cefotaxime prior to discharge B. done mouth-to-mouth resuscitation C. an intern who just inserted IV line D. 302. A 24 year old man reports a painful purulent urethral discharge. Direct gram stain reveal the presence of intracellular gram-negative diplococci. The patient has STI due to: Neisseria meningitides A. Treponema pallidum B. Neisseria gonorrhoeae C. Haemophilus ducreyi D. 303. A. B. C. D.

Your treatment of choice for patient in No. 31 is: Benzathine penicillin Ceftriaxone + Azithromycin Doxycycline Ciprofloxacin

304. A young woman developed a feverish illness with painful swelling of her knee, elbow, & wrist joints. She had a sparse rash on the distal parts of her limbs, consisting of small hemorrhagic pustules on an erythematous base. What does the patient have? Reiter’s syndrome A. Rheumatic fever B. Candidemia C. Disseminated Gonococcal Infection D. Gram positive bacilli 305. Two employees of the microbiology department ordered take-out food from a local Chinese restaurant. Two hours after lunch, one of the individuals experienced severe abdominal cramps, nausea, and vomiting. On questioning, she indicated that the only thing she had eaten that day was fried rice. Which of the following organisms is most likely to have contributed to this illness? Bacillus anthracis A. Corynebacterium diphtheriae B. Listeria monocytogenes C. Bacillus cereus D. 306. A 42 year old, diabetic female, developed fever due to a tooth abscess (left molar). No consultations done & self-medicated with guava leaves. After 2 days, she developed muscle spasms over the left cheek. What is your diagnosis? Rabies A. Tetanus B. Botulism C. temporomaxillary ankylosis D. 307. A. B. C. D.

Your drug of choice for the case in No. 35: Metronidazole Penicillin G Clindamycin Erythromycin

308. Patient had no history of tetanus immunizations before. What vaccine(s) will you give?

A. B. C. D.

Td only Td + TIG None IVIG only

309. A 30 year old male developed abdominal cramps, diarrhea, and fever that presented 8 hours after ingestion of reheated corned beef. What is the causative agent of this food poisoning? Clostridium perfringens A. Staphylococcus aureus B. Bacillus cereus C. Vibrio cholera D. 310. A. B. C. D.

The causative agent of pseudomembranous colitis is: Clostridium tetani Clostridium perfringens Clostridium difficile Clostridium botulinum

311. Which of the following is the mechanism of action of botulinum toxin causing botulism? Blocks the release of inhibitory neurotransmitter glycine and GABA from A. Renshaw cells in spinal cord Blocks the release of acetylcholine resulting in flaccid paralysis B. Cytotoxin kills enterocytes causing pseudomembrane formation C. ADP ribosylation of elongation factor-2 which blocks tRNA translocation D. causing inhibition of host protein synthesis Gram negative bacilli 312. A 73 year old man is diagnosed with malignant otitis externa. organism is the most likely cause of this infection? Klebsiella pneumoniae A. Pseudomonas aeruginosa B. Escherichia coli C. Proteus mirabilis D.

What

313. A 35 year old woman presents with a UTI and nephrolithiasis. The urine has a high pH. What organism is the most likely responsible for this infection? Klebsiella pneumoniae A. Pseudomonas aeruginosa B. Escherichia coli C. Proteus mirabilis D.

314. A 65 year old diabetic man presents to the emergency room with a severe productive cough producing thick bloody sputum resembling a “currant jelly” like appearance. Culture using MacConkey agar reveals pink colonies, with large mucoid colonies on routine laboratory media. Which of the following organisms is most likely responsible for this patient’s pneumonia? Enterobacter cloacae A. Escherichia coli B. Klebsiella pneumoniae C. Pseudomonas aeruginosa D.

315. A. B. C. D.

The most common etiologic agent for acute pyelonephritis is: Enterotoxigenic E coli E coli K1 Enterohemorrhagic E coli P-pili E coli

316. A man developed a feverish illness with a cough. Within a week a small, pink rash appeared on his trunk. The individual lesions blanched on pressure. The WBC count was 4.5 x 10 9/L with a neutropenia. What is the probable diagnosis? measles A. syphilis B. hypersensitivity reaction C. typhoid fever D. 317. A. B. C. D.

The typhoid carrier state is increased in which of the following: Peptic ulcer disease Schistosoma japonicum infection Pancreatitis men

318. A 24 yo man is traveling in SE Asia. He is eating at local restaurants & from street vendors. He now develops high fevers, anorexia, & frequent passage of small-volume stools containing blood, pus, & mucus. This is associated w/ severe abdominal cramps & painful straining (tenesmus) when having bowel movements. What is the most likely pathogen? ETEC A. Salmonella B. Shigella C. Vibrio cholerae D. 319. A. B. C. D.

Cholera presents with the following symptomatology: few, voluminous, non-bloody stools, abdominal pain, fever bloody diarrhea, abdominal pain, fever watery diarrhea, tenesmus, incontinence nausea, vomiting, abdominal cramps, watery diarrhea

320. A 65 yo cigarette smoker w/ a history of hypertension & mild CHF presents to the ER w/ worsening cough, fever, & dyspnea at rest. The illness began 1 week ago w/ fever, muscle aches, abdominal pain, & diarrhea, w/ nonproductive cough developing later that week & rapidly becoming worse. Therapy for w/c atypical organism must be considered in this case? Chlamydia pneumoniae A. Mycoplasma pneumoniae B. Legionella pneumophila C. Aspergillus fumigatus D. Mycobacteria 321. A 50 year old Vietnamese man presents with a chronic bloody sputum, weight loss, and a cavitary lesion on the left upper lobe on CXR. The man has: Aspergillosis A. Tuberculosis B. Paragonimiasis C. Bronchogenic CA D. 322. in:

M. tuberculosis survives best in regions with high oxygen tension found A. B.

apices of the lung long bone metaphyses

C. D.

323. A. B. C. D.

324. A. B. C. D.

lymph nodes adrenals Which of the following anti-TB meds has the earliest bactericidal activity: INH EMB RIF SM The most common site of Pott’s disease in adults: cervical spine upper thoracic lower thoracic and upper lumbar lower lumbar and sacral

325. Which of the following anti-TB drugs have their activity best inside cavities? INH, RIF, Streptomycin A. INH, RIF B. INH, RIF, PZA, EMB C. INH, PZA D.

326. A. B. C. D.

What is the most common extrapulmonary manifestation of tuberculosis? Genito-urinary TB Scrofula TB meningitis TB pericarditis

327. The initial diagnostic procedure to be done in all patients suspected of TB, regardless of symptoms is: CXR A. AFB sputum examination B. TB culture C. PPD D. 328. Before starting short course regimen with INH, RIF, and PZA, the following exam(s) should be requested: ALT and AST A. BUN and Crea B. Uric acid C. ALT, AST, and Uric acid D. 329. The anti-TB drug that interacts with oral contraceptive medications with a risk of decreased protection against pregnancy. INH A. RIF B. PZA C. EMB D. Spirochetes 330. A 28 year old man presents w/ a new genital ulcer on his penis that is painless. He is sexually active & noticed the lesion 1 week ago. The ulcer is 1 cm in size, has an eroded base, & an indurated margin. Dark-field examination of the ulcer fluid confirmed the diagnosis. Primary syphilis A. Secondary syphilis B. Latent syphilis C. Tertiary syphilis D.

331. A 30 year old male, came to an infectious disease specialist because of a positive VDRL, as pre-requisite exam for his application abroad to work as an engineer. Patient is asymptomatic. He has had sexual contact with a commercial sex worker once 6 years ago. What is your most likely diagnosis? Primary syphilis A. Secondary syphilis B. Latent syphilis C. Tertiary syphilis D. 332. His TPHA test is positive for syphilis and quantitative VDRL was 1:16. What is the most adequate treatment for the above patient? Ceftriaxone 125mg IM SD A. Ciprofloxacin 500mg SD + Azithromycin 1g SD B. Benzathine Pen G 2.4 MU IM weekly for 3 doses C. Benzathine Pen G 2.4 MU IM SD D. 333. A syphilitic damage to the posterior columns and dorsal roots of the spinal cord with impairment of position and vibration sense is: Meningitis A. General paresis B. Tabes dorsalis C. Meningovascular syphilis D. 334. A 20 year old, male, came in because of fever, jaundice, severe myalgia, and conjunctival suffusion. 2 days ago, he was cleaning the canal in front of their house. Their house infested with rats. What is the patient’s condition? Dengue fever A. Typhoid fever B. Malaria C. Weil’s syndrome D. 335. The main pathology responsible for the clinical manifestations of the patient in No. 58 is: granulomatous reaction A. immune enhancement B. sequestration C. vasculitis D. 336. Leptospires are eliminated from all sites in the host with the formation of antibodies but they may persist for weeks or months in the: proximal renal tubule A. bladder B. spinal cord C. liver D. 337. Among the available tests for leptospirosis, the most sensitive and specific is: Culture with EMJH A. ELISA for antibodies B. MAT using genus specific antigen L. patoc C. MCAT D. Parasitology 338. A young man returned to Manila from Palawan. A week later, he felt unwell with intermittent fever & headache & treated himself for influenza. 3 days later he as behaving peculiarly & was referred to the hospital. A thin blood smear shows multiple ring forms in a single RBC & ring forms located at the

periphery of the RBCs. About 25% of the cells are parasitized. What antimalarial regimen would you recommend? S-P + CQ + PQ A. Artemether + lumefantrine B. CQ only C. Quinine + Doxycycline + PQ D. 339. A 25 year old , male, with a history of travel to Palawan developed paroxysms of chills, fever, & sweats 2 weeks later. He was diagnosed to have malaria & was treated with chloroquine. He eventually recovered. 6 months later in Manila, he had a relapse of chills, fever, & sweats. What is the etiologic Plasmodium in this case? vivax A. ovale B. malariae C. falciparum D. 340. Which of these anti-malarial drug is essential for eliminating the exoerythrocytic cycle & affecting radical cure? primaquine A. chloroquine B. mefloquine C. quinine D. 341. The following diagnostic test for malaria has the advantage of concentrating parasites, thus increasing diagnostic sensitivity. Thin malarial smear A. Optimal B. Parasight F C. Thick malarial smear D. 342. A. B. C. D.

Red blood cells of all ages can be attacked by: P. Vivax P. ovale P. falciparum P. malariae

343. A 19 yo woman was traveling in a rural area of South America. She returned 3 weeks ago, &, over the past few days, has gradually developed lower abdominal pain & diarrhea. Now the symptoms are much worse w/ 10 stools a day consisting mostly of mucus & blood. She is afebrile, the abdomen is tender in LLQ, & the remaining examination is normal. Her stool is most likely comprised of blood & mucus. Which of the ff. is the most likely causative organism? Salmonella infection A. Shigella infection B. V. parahaemolyticus infection C. E. histolytica infection D. 344. A 35 year old Filipino presents with recurrent fever, headache, photophobia, and painful lymphangitis in his left leg. The best way to diagnose filariasis caused by Wuchereria bancrofti is: biopsy of any inflamed lymph nodes to demonstrate the adult worm A. serologic studies B. observance of intense itching after a single dose of diethylcarbamazine C. demonstration of microfilariae in blood taken between 9PM and 2AM D. 345. After swimming in a freshwater pond in Leyte, a 22 year old man develops intense itching, edema, & a rash. 3 months later, he experiences

abdominal pain, diarrhea, and blood in the stool. Examination of stool revealed eggs with an inconspicuous spine. What organism is most likely responsible for this man’s illness? Balantidium coli A. Entamoeba histolytica B. Schistosoma japonicum C. Hymenolepis nana D. Viral Infections 346. A. B. C. D.

347. A. B. C. D.

Infective stage of Avian flu (H5N1): incubation period prodromal stage lower respiratory stage recovery stage Infective stage of SARS: incubation period prodormal stage lower respiratory stage recovery stage

348. A 19 year old male, was bitten on the leg by a dog, 6 weeks previously; the wound healed normally but 3 days ago he became apathetic, refused food, found difficulty in walking, had neck stiffness and weakness of the lower limbs. The CSF was normal. What is the diagnosis? Paralytic rabies A. Furious rabies B. GBS C. Meningitis D. 349. A. B. C. D.

84. The standard laboratory test for rabies: Seller’s stain for Negri bodies DFA test Virus isolation RT-PCR

350. A 40 year old man presents to you with acute onset of fever, headache and abnormal behavior. CSF analysis shows increased protein, normal glucose and lymphocytic pleocytosis. CT scan shows localizing lesions in the temporal lobe. Further confirmation on EEG shows specific spike-and-dome pattern in the said lobes. The man probably has: Bacterial meningitis A. Herpes encephalitis B. Cryptococcal meningitis C. Viral meningitis D. 351. A 23 yo woman develops vesicular lesions on an erythematous base on her vulvar area. She has tender LADP & dysuria as well. Which of the ff. is the most likely causative organism? CMV A. HSV-2 B. Treponema pallidum C. Varicella zoster D. 352. A 30 year old female returned from an Aedes mosquito infested area in Southeast Asia a week and a half ago and presents with fever, chills, headache, myalgia, and deep bone pain in the back. What virus most likely caused her symptoms?

A. B. C. D.

353. A. B. C. D.

354. A. B. C. D.

Yellow fever virus West Nile virus Dengue virus Hantavirus The pathogenesis of dengue fever is based on: granulomatous reaction immune enhancement sequestration vasculitis Which of the following ways HIV is transmitted perinatally ? Sexual contact Transfer of infected blood Breastfeeding After birth

355. A 52 year old mother of 5 children sees you because she was told that her blood was rejected by the Red Cross due to positive ELISA and an indeterminate WB (a single p24 band) for HIV 3 months ago. She denies risks except for heterosexual sex with 2 male partners during the last 6 months. A second WB 1 month ago showed only a p24 band. Serologic testing at this time is likely to show: HIV-1 infection A. HTLV-1 infection B. HIV-2 infection C. No evidence of retroviral infection D. 356. A. B. C. D.

The most common opportunistic infection in AIDS patients: Tuberculosis Oral Candidiasis Pneumocystis jiroveci pneumonia Cryptococcal meningitis

Fungal Infections 357. A 45 year old male, with CD4 count of 100/L, presents with gradual onset and progression of fever, dry cough, and dyspnea over a month. Laboratory shows an elevated LDH and arterial hypoxemia. CXR reveals bilateral interstitial infiltrates with a ground glass appearance. What possible opportunistic infection does the patient have? TB A. Bacterial pneumonia B. Pneumocystis jiroveci pneumonia C. Histoplasmosis D.

358. A 47 yo heterosexual, edentulous man presents w/ fever, fatigue, swollen cervical nodes, & whitish exudates on tongue, bleeding when scraped off. The patient’s condition is: Pseudomembranous candidiasis (Thrush) A. Chronic hyperplastic candidiasis B. Angular cheilitis C. Erythematous candidiasis D.

Bioterrorism 359. A 58 year old male, hide porter presents to the ERD with fever, headache, and abdominal pain. He had been well until flu-like symptoms started 3 days ago. Shortly after admission, respiratory failure develops and he becomes hypotensive. CXR shows a widened mediastinum & bilateral pleural effusions. Thoracentesis shows bloody pleural fluid. Which of the following diagnostic studies will confirm the diagnosis? Culture of blood for bacteria A. Nasal swab culture B. Serologic testing C. Pleural biopsy for histologic examination and mycobacterial culture D. PULMO : 360. Which of the following microbial pathogens almost never cause pulmonary cavities? Mycoplasma pneumoniae A. Oral anaerobes B. S. aureus C. S. pneumoniae serotype III D. 361. A 40 y/o female diagnosed to have community acquired pneumonia. She is a known diabetic with good blood glucose control for the last 3 months. What is the risk classification of this patient? Minimal risk C. Moderate risk A. Low risk D. High risk B. 362. A. B.

What is the most commonly involved extra pulmonary site in tuberculosis? Bones C. Lymph nodes Genitourinary tract D. Meninges

363. Anna is a health center nurse assigned to TB patients under the DOTS. She has a (+) AFB smear, & she is asymptomatic. What is her PTB diagnostic classification? A. 1 C. 3 B. 2 D. 4 364.

What is the recommended treatment regimen for PTB Class 2? A. No treatment; observe C. Secondary prophylaxis B. Primary prophylaxis D. Six months anti-Koch’s therapy

365.

The most common of the stimuli that evoke acute exacerbations of asthma A. Aspirin C. Respiratory infection B. Beta blocker D. Exercise

366. A 30 year old female was diagnosed to have bronchial asthma. She claims to have night time symptoms 7 x / weeks & PEFR = 70%. What is the severity of the asthma? A. Mild intermittent C. moderate persistent B. Mild persistent D. Severe persistent 367. A. B. C. D.

The most common route for bacterial pneumonia Aerosolization Microaspiration Hematogenous spread Direct spread from contigous infected site

368. A 60 year old male, 35 pack years smoker came for consult because of chronic cough. PE was normal & also with the chest x-ray. On spirometry FEV1 / FVC = 65 & FEV1 = 60. What is the COPD severity? A. Mild C. Severe B. Moderate D. Very severe 369.

The most common cause of pleural effusion. A. Left ventricular failure C. Pneumonia B. PTB D. Lung cancer

370. A. B.

371. A. B.

372. A. B.

The predominant cells affected in COPD pathogenesis are: Eosinophils C. Neutrophils T-lymphocytes D. Mast cells The form of emphysema that is related to alpha-1-antitrypsin deficiency. Distal acinar C. Paraseptal Panacinar D. Centriacinar The usual cause of impaired gas exchange in patients with COPD/ is: Shunt C. Ventilation perfusion mismatch Impaired diffusion D. Hypoventilation

373. A patient with bronchogenic CA was noted to have miosis, ptosis, anhidrosis of the right side of face. This syndrome is called: Superior vena cava C. Pancoast’s A. Horner’s D. Eaton-Lambert B. 374. A known COPD patient was admitted because of fever, purulent sputum & severe dyspnea. He was noted to be lethargic. ABG revealed a ph=7.31 PaCO2=65 mmHg, PaO2=55 mmHg, HCO3=23, O2 sat 88%, which is interpreted as: Acute respiratory acidosis with hypoxemia A. Acute respiratory alkalosis with hypoxemia B. Acute metabolic acidosis with hypoxemia C. Acute metabolic alkalosis with hypoxemia D. 375. is:

The earliest demonstrable mechanical defect in young cigarette smokers A. B. C. D.

Obstruction of small airways Hypertrophy and hyperplasia of the bronchi Increase in mucus production Restrictive ventilatory impairment

376. The most common form of lung cancer arising in lifetime nonsmokers, in women, and in young patients 45 years old) is:( Adenocarcinoma C. Large cell A. Small cell D. Squamous cell B. 377. A. B. C. D.

378. A. B.

Which of the following pathologic findings best describe emphysema? Increased number of goblet cells Conversion of ciliated respiratory epithelium to cuboidal epithelium Hypertrophy of bronchial mucosa Imbalance in the protease – antiprotease system A lung volume abnormality that signifies hyperinflation High total lung capacity C. Low residual volume High diffusion lung capacity D. Low functional residual capacity

379. The most common acid-base abnormality during acute exacerbations of asthma Respiratory alkalosis C. Metabolic acidosis A. Respiratory acidosis D. Metabolic alkalosis B.

380. The single most important factor that affects favorable outcome of COPD patients Smoking cessation C. Regular use of inhaled bronchodilators A. Pulmonary rehabilitation D. Oxygen therapy B.

381. A. B.

382. A. B. C. D.

383. A. B. C. D.

384. A. B.

Which of the following factors best influence the transmission of PTB? Occupation C. Bacillary load Body weight D. Nutritional status When should you consider drug resistance in PTB? Positive tuberculin test after treatment Unchanged chest x-ray findings after 2 months of treatment Positive sputim AFB after 4-6 minutes of treatment Positive sputum AFB after 2 months of treatment One of the following is an indication for chemoprophylaxis for PTB: Persons recently infected with mycobacterium tuberculosis Contacts of patients treated for PTB Tuberculin-negative persons with stable CXR findings Patients with diabetes mellitus Which of the following is a primary tumor of the pleura? Adenocarcinorma C. Lymphoma Mesothelioma D. Lipoma

385. Which of the following conditions is not a cause of exudative pleural effusion? A. Neoplastic diseases C. Collagen vascular diseases B. Tuberculosis D. Nephrotic Syndrome 386.

Which of the following conditions exudative pleural effusion? A. Congestive heart failure C. Liver cirrhosis B. Nephrotic syndrome D. Tuberculosis

387. The following is an indication for tube thoracostomy insertion in parapneumonic effusion: A. Pleural fluid pH > 7.20 C. Pleural fluid LDH < 1000 IU/L B. Presence of gross pus D. Pleural fluid glucose > 50 mg/dl 388. A. B. C. D. 389.

The predominant inflammatory cell in COPD: A. Eosinophils C. Macrophages B. Neutrophils D. Lymphocytes

390. A. B. C. D. 391.

The most common site of PTB: Anterior segment of upper lobe Posterior segment of upper lobe Basal segment of lower lobe Inferior segment of lower lobe

The most common route for the development of pneumonia is : Aspiration Hematogenous Contiguity Direct inoculation

A patient who is edentulous would unlikely develop pneumonia secondary to : A. Streptococcus pneumoniae B. Mycoplasma pneumoniae

C. Hemophilus influenzae D. Anaerobes

392. Important laboratory work-ups in the diagnosis of pneumonia include the following EXCEPT: A. Chest X-Ray B. CBC C. Sputum AFB D. Sputum Gram stain & Culture Sensitivity 393. Which of the following factors will make you decide to hospitalize a patient with pneumonia? A. He is 60 years old B. He has concomitant PTB C. He has osteoarthritis D. He cannot take oral medications 394. Atypical pneumonia is being considered in a young patient; the antibiotic of choice is : A. Penicillin B. Macrolide C. Co-amoxiclav D. Qinolones 395. A. B. C. D.

Which is not considered a risk factor for PTB? Age Sex Genetics Environment

396. The Chest X-ray reported small discrete tubercles scattered in both lungs more on the bases, you will consider: A. Primary complex B. MDR-TB C. Miliary TB D. Scrofula 397. In a patient with cervical lymphadenopathies, aside from TB the differential diagnosis is : A. Lymphoma B. Bronchogenic Carcinoma C. None of the above D. All of the above 398. A. B. C. D.

Pedro Taong-Bato was diagnosed as PTB Class 3, defined as : TB suspect TB infection TB clinically active TB not clinically active

A. B. C. D.

Which of the following is a second line anti TB drug? Para-aminosalicylic acid Rifampicin Isoniazid Streptomycin

399.

400.

The most common cause of COPD is :

A. B. C. D.

Cigarette smoking Air pollution Indoor pollution Respiratory infection

ENDOCRINE : DR. ROLANDO LIGON 401. A. B. C. D.

402. A. B.

403.

All of the following features are seen in type 1 B DM EXCEPT: Tendency to develop ketosis Presence of immunologic markers for beta cell destructions Absolute requirement for insulin therapy Can develop at any age All of the following conditions can cause testing hypoglycemia EXCEPT: Insulinoma c. Mesothelioma Addison’s disease d. Alimentary hypoglycemia

A 49 years old female had a very strong family history of DM. Her FBG is 107 mg/dl. All the other findings are risk factors for the development of type 2 on EXCEPT: BMI of 28 kg/m2 c. HDL level of 33 mg/dl A. Diastolic pressure of 90 mmHg d. Triglycerides level of 200 mg/dl B.

404. A 54 year old presented with polyuria, polyphagia, polydepsia and weight loss. All of the following tests can be used to make the diagnosis of DM except: RBS c. Two hour plasma glucose after a 75 g glucose load A. FPG d. HbAiC B. 405. A 51 years old male presented with symptomatic hypoglycemia following streneus activity. An unresectable Insulinoma was documented by MRI scan. The drug that can be used to treat hypoglycemia in this setting is: Prednisone c. Plenytoin A. Diazozide d. Propranolol B. 406. An 18 years old boy presented with severe abdominal pain associated with acetone breath. RBS was noted to be 600 mg/dl. Other biochemical parameters that confirms the diagnosis of DKA are as follows EXCEPT: HCO3 of 10 meq/L c. Plasma osmolality of 310 mosm/L A. Arterial pH of 7.5 d. Wide anion gap B. 407. An 18 years old filipina G1P0 10 weeks AOG consulted an obstetrician for regular prenatal check-up. She denied family history of diabetes and previous abortion. Her BMI was 20 kg/m2. During her initial visit, it is a must to do: FPG c. OGTT A. GCT d. HbAic B. 408. A 54 years old female was noted to be diabetic for more than 30 years. Nutritional recommendation for her are as follows EXCEPT: 25 g/day of fibers c. 10% of Kcal/day of protein A. < 300 mg/day of cholesterol d. 30% of Kcal/day of polyunsaturated fats B. 409. A 22 years old female presented with a grade II goiter causing venous distention over the neck and difficulty of breathing especially when the arms are raised. This is known as: Mobious sign c. Pemberton’s sign A. Jod-Basedow effect d. Wolff-Charkoff effect B.

410. A. B.

The most common type of thyroid CA is: Papillary c. Medullary Follicular d. Ana plastic

411. A 17 year old female presented with a solitary module of the thyroid gland; no S/SX of hypo or hypothyroidism was noted FNAB of the said nodule did not disclosed a malignant process. Thyroid ultrasonography revealed a cystic lesion on the right lobe of the thyroid gland. The therapeutic option that can be given to this particular case is: Suppression therapy c. Radioactive iodine ablation A. Thyroid sclerotherapy d. Total thyroidectomy B. 412. A 44 years old female presented with uncontrolled hyperthyroidism associated with seizures and unexplained jaundice in the setting of severe pneumonia. All of the following agents can be given to her EXCEPT: Dexamethasone c. Propranolol A. SSKI d. Methimazole B. 413. A 55 years old female from Benguet presented with multinodular goiter and classical signs and symptom of hypothyroidism. Her TSH level is markedly elevated. The only therapeutic option recommended for her is: Levothyroxine therapy c. Thyroglobulin therapy A. Liothyronine therapy d. Iodine supplementation B. 414. A 23 years old female presented with sudden onset of a painful goiter and subtle manifestations of thyrotoxicosis. The test that will differentiate it from Graves disease is: TSH c. T3 / T4 A. 24 hours RAIU d. Thyroid ultrasound B. 415. A. B.

The thyroid hormone that has the highest metabolic potency is: Monoiodothyrosine c. Triiodothyronine Diiodothyrosine d. Tetraiodothyronine

416. A 45 years old male was admitted at ICU because of AMI associated with atrial fibrillation. The thyroid gland was not enlarged; TSH is slight elevated with depressed T3 and T4 levels. Such findings are consistent with: Primary hypothyroidism c. Secondary hypothyroidism A. Sick euthyroid syndrome d. Secondary hypothyroidism B. 417. A. B.

418. A. B.

The most common manifestation of Cushing’s syndrome is: Hypertension c. Centripetal obesity Weight gain d. Proximal myopathy The most common cause of endocrine hypertension is: Primary aldosteronism c. Pheochromocytoma Cushing’s syndrome d. Congenital adrenal hyperplasia

419. A 54 year old male presented with cushingoid fescie associated with hypertension. The most sensitive test to confirm your clinical diagnosis is: Serum cortisol (PM) A. 24 hours urinary free cortisol B. Low dose dexamethasone suppression test C. High dose dexamethasone suppression test D. 420. A 34 years old male presented with the triad of HA, dizziness and palpitations with paroxysmal hypertension. VMA studies are markedly elevated on two separate occasions. All of the following drugs can be used initially to control the hypertension in this setting EXCEPT:

A. B.

Alpha blockers Beta blockers

c. ACE inhibitors d. Calcium blockers

421. A 23 years old male presented with severe abdominal pain and behavioral changes. He had previous history of open craniotomy 10 years ago craniopharyngioma. The best screening test to confirm your diagnosis is: Cosyntropin test c. Clonidine test A. Glucagons test d. Saline infusion test B. 422. A 53 years old male presented with classical features of cushing’s syndrome. Serum cortisol taken at PM was markedly elevated. The test that will differentiate whether the lesion is in the pituitary or in the adrenal is: Low dose dexamethasone test A. High dose dexamethasone tes B. 24 hours urinary free cortisol C. cosyntropin test D. 423. The pituitary hormone in which the predominant central control mechanism is inhibitory: Thyrotropin c. Prolactin A. Adenocorticotropin d. Gonadotropin B. 424. A 53 years old female presented with progressive weight gain characterize as truncal obesity with moon fascie and facial plethora. High dose dexamethasone test was suppressed by 50% from baseline urine cortisol lever. The lesion was confirmed by MRI seen. The treatment of choice in this setting is: Open craniotomy c. Transphenoidal excision A. Bromocriptine therapy d. Irradiation B. 425. A 23 years old female presented with amenorrhea and galactorrhea. She consulted your clinic because of her desire to have a baby. Her PRL was about 75 mg/L. CT scan of the pituitary revealed an 8 mm sellar mass. The treatment of choice for her case is: Bromocriptine therapy c. Transphenoidal excision A. Open craniotomy d. Irradiation B. 426. A. B.

427. A. B.

428. A. B.

All of the following are causes of central diapidus insipidus EXCEPT: SLE c. Sheehan’s syndrome Hypercalcemia d. Breast CA The enzyme responsible for the conversion of testosterone to DHT is: Aromatase c. 17-20 lyase 5 alpha reductase d. Sulfokinase The most potent estrogen is: Estradiol Estrone

c. Estriol d. Catechol estrogens

429. A 35 years old female diagnosed to have terminal stage of breast CA was admitted in the hospital due to severe dehydration. The serum calcium was markedly elevated on two separate occasion. The paracrine factor that is elevated in this particular setting is: PTH mid region c. PTH amino terminal A. PTH carboxyl end d. PTH related peptide B. 430. All of the following conditions are associated with PTH ineffective mechanism of hypocalcemia EXCEPT: Pseudohypoparathyroidism c. Anticonvulsant therapy A. Chronic renal failure d. Severe hypomagnesemia B.

431. A. B.

432. A. B.

An index of glucose control for the past 12 weeks is: HbA1C c. OGTT Fructosamine d. FBS A test which is a reflection of hepatic glucose production: HbA1C c. OGTT Fructosamine d. FBS

433. A 32-year-old female was noted to have GDM on her present pregnancy. The test that should be done to her 6 weeks after delivery is: HbA1C c. 2-hr OGTT A. Fructosamine d. FBS B. 434. A 57-year-old male presented with 3 P’s and weight loss. BMI was 35 2 kg/m ,BP of 140/90 and background retinopathy on fundoscopic exam. The FBS and C-peptide levels are markedly elevated. The hallmark of this particular type of diabetes is: Presence of markers of autoimmunity A. Independent on insulin for survival B. Associated with genetic defect of insulin secretions C. Obesity is always present D. 435. An 18-year-old female complained of vaginal pruritus. She had a strong family history of DM. Her FBS was 122 mg/dl. The test that will confirm the biochemical diagnosis of DM is: HbA1C c. Urine sugar A. GCT d. 2-hr OGTT B. 436. A. B.

The most abundant active thyroid hormone is: MIT c. T3 DIT d. T4

437. The following metabolic effects are increased by thyroid hormones EXCEPT: Bone resorption A. Glucocorticoid degradation B. Cathecholamine receptors C. Superoxide dismutase levels D. 438. A 43-year-old female underwent subtotal thyroidectomy for DTG. Ten years later, she complained of progressive weight gain, cold intolerance and constipation. The most sensitive test to determine adequacy of treatment is: Thyroglobulin c. Free T3 A. TSH d. Free T4 B. 439. A 23-year-old pregnant mother presented with sudden onset of a nodular goiter and subtle s/sx of thyrotoxicosis associated with fever. The following tests can be done in her case EXCEPT: Thyroid ultrasonography c. 24-hr RAIU A. TSH d. Free T3 / T4 B. 440. A 55-year- old male underwent total thyroidectomy and subsequent RAI ablation due to follicular thyroid CA. the test that can be used to monitor activity of his condition is: Thyroid scan c. FNAB A. Thyroglobulin d. TSH B. 441. A. B.

The following manifestations are seen in hypertyroidism EXCEPT: Diastolic hypertension c. Plummer’s nail Infertility d. Lid lag

442. A. B.

The following manifestations are seen in hypothyroidism EXCEPT: Psychosis c. Pretibial myxedema Heart failure d. Somnolence

443. A 32-year-old female presented with uncontrolled thyrotoxicosis, clouded sensorium, and tachycardia unproportional to the fever in the setting of a diffuse goiter. The following drugs can be given to her EXCEPT: Carbimazole c. Lugol’s solution A. Dexamethasone d. Propranolol B. 444. A 40-year-old diabetic noticed a nodular goiter associated with easy fatigability and hoarseness. T4 is depressed while TSH is markedly elevated. The only therapeutic option recommended to her case is: PTU c. Iodine A. Levothyroxine d. Liothyronine B. 445. A 34- year-old male diagnosed to have acromegaly complained of a grade II goiter. The mechanism of the pathogenesis of his goiter is: Thyroid autoregulation c. IGF-1 stimulation A. TSH stimulation d. Autoimmunity B. 446. The following steroids are produced in the zona fasciculata of the adrenal cortex EXCEPT: Pregnenolone c. Cortisol A. Aldosterone d. Testosterone B. 447. A. B.

The most abundant cathecholamine produced by the adrenal medulla is: L-dopa c. Norepinephrine Dopamine d. Epinephrine

448. A 23-year-old female complained of cushingoid features. The most reliable screening test for her is: Serum cortisol (AM) c. 24-hr urinary free cortisol A. Serum cortisol (PM) d. Low dose dexamethasone test B. 449. A 33-year-old nurse presented with the triad of headache, palpitations, and diaphoresis in the setting of paroxysmal hypertension. The test that could induce above features is: Clonidine test c. Saline infusion test A. Glucagon test d. Glucose suppression test B. 450. A 55-year-old female showed a 50% suppression of baseline urinary cortisol level after a 2-day high dose dexamethasone test. The localizing procedure that will confirm the diagnosis is: CT scan of the adrenals c. MIBG of the adrenals A. MRI of the adrenals d. CT scan of the pituitary B. 451. A. B.

The most common cause of spontaneous primary Addison’s disease is: PTB c. AIDS Autoimmune adrenalitis d. Abrupt withdrawal of steroids

452. The following conditions are associated with normal levels of catecholamines EXCEPT: Hyperthyroidism c. Hypertensive encephalopathy A. Menopause d. Hypoglycemia B. 453. A 34-year-old male was noted to have spontaneous hypokalemia with inappropriate kaliuresis. His plasma rennin activity was suppressed, aldosterone assay was high, and the CT scan of the adrenals showed bilateral adenomas. The treatment of choice for him is: Bilateral adrenalectomy c. TSE of the pituitary A.

B.

Captopril therapy

d. Spirinolactone therapy

454. A 25-year-old female presented with headache, sweating, and palpitations with sustained hypertension. VMA studies were high. The following agents can be initially used to control the hypertension EXCEPT: Nifedipine c. Prazocin A. Captopril d. Propranolol B. 455. A 21-year-old asthmatic has been taking 20 mg/day of prednisone for 2 years. He was free of attacks for the last 3 months, so he stopped the drug abruptly. 3 days later, he was seen at the ER because of refractory hypotension unresponsive to fluids and vasopressor agent. The following lab abnormalities are consistent with his condition EXCEPT: Hyponatremia c. Hypokalemia A. Hypoglycemia d. Hypercalcemia B. 456. A. B.

The most common cause of pituitary macroadenoma is: PRL-secreting adenoma c. ACTH-secreting adenoma GnH-secreting adenoma d. GH-secreting adenoma

457. The recommended treatment of choice for prolactinoma measuring 20 mm in diameter is: Irradiation c. Open craniotomy A. Transphenoidal excision d. Bromociptine therapy B. 458. A 34-year-old teacher presented with progressive coarsening of facial features, acral enlargement, frontal bossing. The test that will confirmed the clinical diagnosis is: Single GH assay c. PRL assay A. FSH/LH assay d. Glucose suppression test B. 459. An 18-year-old male underwent open craniotomy for craniopharyngioma. On his 2nd postop day, he developed massive polyuria. The test that will differentiate it from nephrogenic DI is: Dehydration test c. Vasopressin test A. Saline infusion test d. Plasma osmolality test B. 460. A 55-year-old male diagnosed to have stage IV bronchogenic CA presented with polyuria, hyponatremia and hypoosmolality. The most likely diagnosis is: Primary polydipsia c. Nephrogenic DI A. SIADH d. Central DI B. ONCOLOGY : DR. ROSELLE DE GUZMAN 461. This supraclavicular lymph node is enlarged in gastrointestinal malignancies Blummer’s node C. Stellar node A. Virchow’s node D. Sister Mary Joseph’s node B. 462. A. B.

This periumbilical nodes enlarges in intraabdominal malignancies Blummer’s node C. Stellar node Virchow’s node D. Sister Mary Joseph’s node

463. This syndrome of erythematosus, painful plaques, with fever and neutrophilia found in hematologic and gastrointestinal carcinomas is called Melanogastroacanthosis syndrome C. Gastronomic syndrome A. Sweet’s syndrome D. Leser-Trelat sign B.

464. Gastric adenocarcinoma can produce gray-browm symmetric velvety plaques commonly on the neck, axilla, flexor areas and anogenital areas. This is called: Tripe palms C. Acanthosis nigricans A. Bazex’s disease D. Leser-Trelat sign B. 465. A. B.

A patient who has pernicious anemia is predisposed to have: Duodenal cancer C. Pancreatic cancer Colon cancer D. Gastric cancer

466. Hepatitis C may predispose a patient to develop Hepatocellular cancer. When he develops liver cancer this tumor marker is markedly elevated Alpha-fetoprotein C. Gamma-globulins A. Beta- HCG D. Delta carcinoantigen B. 467. In pancreatic cancer, CA 19-9 is elevated. Patient may also present with distention of the gall bladder, this sign is called Jean Collique sign C. Leser-Trelat sign A. Courvoiser’s sign D. Tripe Palm’s sign B. 468. Hepatitis B patients may cause Hepatocellular cancer. Exposure to this toxin may also lead to Hepatocellular cancer Asbestos C. Aflatoxin A. Lye D. Digitoxin B. 469. A. B.

470. A. B.

471. A. B.

472. A. B.

Brick red stools in large intestinal cancer is more associated with Right-sided colon cancer C. Rectal cancer Left-sided colon cancer D. Transverse colon cancer Mucoid and Bright-Red stool is most commonly found in patients with Right-sided colon cancer C. Rectal cancer Left-sided colon cancer D. Transverse colon cancer Right sided colon cancer do not present with: Abdominal pain C. Tenesmus Vomiting D. Obstruction The type of anemia associated with cancer is: Iron deficiency anemia C. Hemolytic anemia Sickle cell anemia D. Pernicious anemia

473. A 60 year old male, presented with easy fatigability and chest pain. On PE he was pale but no abdominal mass was palpated. History revealed patient had a Bilroth II procedure 15 years ago due to a perforated peptic ulcer. The patient acutely is having several bouts of vomiting especially after a heavy meal. Patient most likely is suffering from: Gastric cancer C. Rectal CA A. Hepatoma D. Pancreatic cancer B. 474. Peutz-Jegher’s syndrome, Gardner’s syndrome, and FAP coli predispose a person to: Gastric CA C. Renal CA A. Pancreatic CA D. Colonic CA B. 475. A. B.

Painless jaundice and elevated CA 19-9 strongly suggest: Dueodenal CA C. Pancreatic CA Carcinomatoid CA D. Renal CA

476. A 39 year old male, known Hepatitis B carrier who is fond of eating peanuts, developed right upper quadrant pain and weight loss. What laboratory exam would you request to assist you in the diagnosis? Chest x-ray C. Barium enema A. Endoscopy D. Ultrasound of the hepatobiliary tree B. 477. If there is a mass found in the RUQ, what blood exam can help you in making a definite diagnosis? CEA C. CA – 125 A. Alpha fetoprotein D. B – HCG B. 478. Gastric CA is differentiated from gastric lymphoma by the following procedure: UGIS C. Endoscopy and biopsy A. Endoscopy D. Plain abdominal radiography B. 479. A. B.

480. A. B.

481. A. B.

482. A. B.

483. A. B.

484. A. B.

485. A. B.

486. A. B. C. D.

487. A. B. C. D.

488. A. B.

Pernicious anemia with early satiety may suggest: Pancreatic CA C. Right sided colonic CA Linnitis plastica D. Left sided colonic CA Intestinal obstruction is more common in: Right sided colon CA C. Pancreatic CA Left sided colon CA D. Renal CA The leading primary tumor site in all ages: Colorectal C. Lung Lymphoma D. Liver The leading primary tumor site in men: Lung C. Liver Pancreas D. Colorectal The leading cancer death is from Breast cancer C. Lung Cancer Liver cancer D. Colorectal cancer The major determinant of treatment outcome: Histologic type C. Physiologic reserve Tumor burden D. Age The karnofsky performance status associated with poor prognosis: >100 C.
View more...

Comments

Copyright ©2017 KUPDF Inc.
SUPPORT KUPDF